GATE Exam 2023 Humanities & Social Sciences (XH) Question Paper With Answer Key

GATE-2023

XH : Humanities & Social Sciences

General Aptitude

Q.1 – Q.5 Carry ONE mark each.

1. Rafi told Mary, “I am thinking of watching a film this weekend.”

The following reports the above statement in indirect speech:

Rafi told Mary that he _______ of watching a film that weekend.

(A)  thought

(B)  is thinking

(C)  am thinking

(D)  was thinking

Answer: (D)

2. Permit : _______ : : Enforce : Relax

(By word meaning)

(A)  Allow

(B)  Forbid

(C)  License

(D)  Reinforce

Answer: (B)

3. Given a fair six-faced dice where the faces are labelled ‘1’, ‘2’, ‘3’, ‘4’, ‘5’, and ‘6’, what is the probability of getting a ‘1’ on the first roll of the dice and a ‘4’ on the second roll?

(A)  1/36

(B)  1/6

(C)  5/6

(D)  1/3

Answer: (A)

4. A recent survey shows that 65% of tobacco users were advised to stop consuming tobacco. The survey also shows that 3 out of 10 tobacco users attempted to stop using tobacco.

Based only on the information in the above passage, which one of the following options can be logically inferred with certainty?

(A)  A majority of tobacco users who were advised to stop consuming tobacco made an attempt to do so.

(B)  A majority of tobacco users who were advised to stop consuming tobacco did not attempt to do so.

(C)  Approximately 30% of tobacco users successfully stopped consuming tobacco.

(D)  Approximately 65% of tobacco users successfully stopped consuming tobacco.

Answer: (B)

5. How many triangles are present in the given figure?

(A)  12

(B)  16

(C)  20

(D)  24

Answer: (C)

Q.6 – Q.10 Carry TWO marks Each

6. Students of all the departments of a college who have successfully completed the registration process are eligible to vote in the upcoming college elections. However, by the time the due date for registration was over, it was found that suprisingly none of the students from the Department of Human Sciences had completed the registration process.

Based only on the information provided above, which one of the following sets of statement(s) can be logically inferred with certainty?

(i) All those students who would not be eligible to vote in the college elections would certainly belong to the Department of Human Sciences.

(ii) None of the students from departments other than Human Sciences failed to complete the registration process within the due time.

(iii) All the eligible voters would certainly be students who are not from the Department of Human Sciences.

(A)  (i) and (ii)

(B)  (i) and (iii)

(C)  only (i)

(D)  only (iii)

Answer: (D)

7. Which one of the following options represents the given graph?

(A)  f(x) = x22|x|

(B)  f(x) = x 2|x|

(C)  f(x) = |x|2x

(D)  f(x) = x2x

Answer: (B)

8. Which one of the options does NOT describe the passage below or follow from it?

We tend to think of cancer as a ‘modern’ illness because its metaphors are so modern. It is a disease of overproduction, of sudden growth, a growth that is unstoppable, tipped into the abyss of no control. Modern cell biology encourages us to imagine the cell as a molecular machine. Cancer is that machine unable to quench its intial command (to grow) and thus transform into an indestructible, self-propelled automaton.

[Adapted from The Emperor of All Maladies by Siddhartha Mukherjee]

(A)  It is a reflection of why cancer seems so modern to most of us.

(B)  It tells us that modern cell biology uses and promotes metaphors of machinery.

(C)  Modern cell biology encourages metaphors of machinery, and cancer is often imagined as a machine.

(D)  Modern cell biology never uses figurative language, such as metaphors, to describe or explain anything.

Answer: (D)

9. The digit in the unit’s place of the product 3999 × 71000 is ________.

(A)  7

(B)  1

(C)  3

(D)  9

Answer: (A)

10. A square with sides of length 6 cm is given. The boundary of the shaded region is defined by two semi-circles whose diameters are the sides of the square, as shown.

The area of the shaded region is _______ cm2.

(A)  6π

(B)  18

(C)  20

(D)  9π

Answer: (B)

Reasoning and Comprehension (XH-B1)

XH-B1: Q.11 – Q.17 Carry ONE mark Each

11. Which word below best describes the idea of being both Spineless and Cowardly?

(A)  Pusillanimous

(B)  Unctuous

(C)  Obsequious

(D)  Reticent

Answer: (A)

12. Choose the right preposition to fill up the blank:

The whole family got together ___ Diwali

(A)  of

(B)  at

(C)  in

(D)  till

Answer: (B)

13. Select the correct option to fill in all the blanks to complete the passage:

The (i) _______ factor amid this turbulence has been the (ii) ________ of high-octane, action-oriented films such as RRR, K.G.F: Chapter 2 and Pushpa from film industries in the south of the country. Traditionally, films made in the south have done well in their own (iii) _________. But increasingly, their dubbed versions have performed well in the Hindi heartland, with collections (iv) ________ those of their Bollywood counterparts.

(A)  (i) disheartening (ii) failure (iii) channels (iv) matching

(B)  (i) redeeming (ii) outperformance (iii) geographies (iv) eclipsing

(C)  (i) shocking (ii) underperformance (iii) cinemas (iv) below

(D)  (i) humbling (ii) bombing (iii) theatres (iv) falling behind

Answer: (B)

14. The following passage consists of 6 sentences. The first and sixth sentences of the passage are at their correct positions, while the middle four sentences (represented by 2, 3, 4, and 5) are jumbled up.

Choose the correct sequence of the sentences so that they form a coherent paragraph:

(1) Most obviously, mobility is taken to be a geographical as well as a social phenomenon.

(2) Much of the social mobility literature regarded society as a uniform surface and failed to register the geographical intersections of region, city and place, with the social categories of class, gender and ethnicity.

(3) The existing sociology of migration is incidentally far too limited in its concerns to be very useful here.

(4) Further, I am concerned with the flows of people within, but especially beyond, the territory of each society, and how these flows may relate to many different desires, for work, housing, leisure, religion, family relationships, criminal gain, asylum seeking and so on.

(5) Moreover, not only people are mobile but so too are many ‘objects’.

(6) I show that sociology’s recent development of a ‘sociology of objects’ needs to be taken further and that the diverse flows of objects across societal borders and their intersections with the multiple flows of people are hugely significant.

(A)  3, 2, 5, 4

(B)  2, 3, 4, 5

(C)  5, 4, 3, 2

(D)  4, 2, 5, 3

Answer: (B)

15. The population of a country increased by 5% from 2020 to 2021. Then, the population decreased by 5% from 2021 to 2022. By what percentage did the population change from 2020 to 2022?

(A)  -0.25%

(B)  0%

(C)  2.5%

(D)  10.25%

Answer: (A)

16. The words Thin: Slim: Slender are related in some way. Identify the correct option(s) that reflect(s) the same relationship:

(A)  Fat: Plump: Voluptuous

(B)  Short: Small: Petite

(C)  Tall: Taller: Tallest

(D)  Fair: Dark: Wheatish

Answer: (A, B)

17. A pandemic like situation hit the country last year, resulting in loss of human life and economic depression. To improve the condition of its citizens, the government made a series of emergency medical interventions and increased spending to revive the economy. In both these efforts, district administration authorities were actively involved.

Which of the following action(s) are plausible?

(A)  In future, the government can make district administration authorities responsible for protecting health of citizens and reviving the economy.

(B)  The government may set up a task force to review the post pandemic situation and ascertain the effectiveness of the measures taken.

(C)  The government may set up a committee to formulate a pandemic management program to minimize losses to life and economy in future.

(D)  The government may take population control measures to minimize pandemic related losses in future.

Answer: (B, C)

XH-B1: Q.18 – Q.26 Carry TWO marks Each

18. Six students, Arif, Balwinder, Chintu, David, Emon and Fulmoni appeared in the GATE-XH exam in 2022. Balwinder scores less than Chintu in XH-B1, but more than Arif in XH-C1. David scores more than Balwinder in XH-C1, and more than Chintu in XH-B1. Emon scores less than David, but more than Fulmoni in XH-B1. Fulmoni scores more than David in XH-C1. Arif scores less than Emon, but more than Fulmoni in XH-B1. Who scores highest in XH-B1?

(A)  Fulmoni

(B)  Emon

(C)  David

(D)  Chintu

Answer: (C)

19. Select the correct relation between E and F.

(A)  E > F

(B)  E < F

(C)  E = F

(D)  E < -F

Answer: (B)

20. A code language is formulated thus:

Vowels in the original word are replaced by the next vowel from the list of vowels, A-E-I-O-U (For example, E is replaced by I and U is replaced by A). Consonants in the original word are replaced by previous consonant (For example, T is replaced by S and V is replaced by T).

Then how does the word, GOODMORNING appear in the coded language?

(A)  HUUFNUSPOPH

(B)  FIICLIQMEMF

(C)  FUUCLUQMOMF

(D)  HEEDATTACRH

Answer: (C)

21. The stranger is by nature no “owner of soil” — soil not only in the physical, but also in the figurative sense of a life-substance, which is fixed, if not in a point in space, at least in an ideal point of the social environment. Although in more intimate relations, he may develop all kinds of charm and significance, as long as he is considered a stranger in the eyes of the other, he is not an “owner of soil.” Restriction to intermediary trade, and often (as though sublimated from it) to pure finance, gives him the specific character of mobility. If mobility takes place within a closed group, it embodies that synthesis of nearness and distance which constitutes the formal position of the stranger. For, the fundamentally mobile person comes in contact, at one time or another, with every individual, but is not organically connected, through established ties of kinship, locality, and occupation, with any single one.

What assumptions can be made about the stranger from the passage above?

(A)  The stranger can become an owner of soil through developing all kinds of charm in more intimate relations.

(B)  The stranger cannot become an owner of soil either in the physical or psychological sense.

(C)  The stranger can become an owner of soil through establishing ties of kinship and so on.

(D)  The stranger might become an owner of soil in the physical sense but not in the psychological

Answer: (B)

22. L is the only son of A and S. S has one sibling, B, who is married to L’s aunt, K. B is the only son of D. How are L and D related?

Select the possible option(s):

(A)  Grandchild and Paternal Grandfather

(B)  Grandchild and Maternal Grandfather

(C)  Grandchild and Paternal Grandmother

(D)  Grandchild and Maternal Grandmother

Answer: (B, D)

23. Five segments of a sentence are given below. The first and fifth segments are at their correct positions, while the middle three segments (represented by 2, 3, and 4) are jumbled up. Choose the correct order of the segments so that they form a coherent sentence:

(1) Consumed multitudes are jostling and shoving inside me

(2) and guided only by the memory of a large white bedsheet with a roughly circular hole some seven inches in diameter cut into the center,

(3) clutching at the dream of that holey, mutilated square of linen, which is my talisman, my open-sesame,

(4) I must commence the business of remaking my life from the point at which it really began,

(5) some thirty-two years before anything as obvious, as present, as my clock-ridden, crime-stained birth.

(A)  2 – 3 – 4

(B)  3 – 2– 4

(C)  4 – 2– 3

(D)  4 – 3 – 2

Answer: (A)

24. “I told you the truth,” I say yet again, “Memory’s truth, because memory has its own special kind. It selects, eliminates, alters, exaggerates, minimizes, glorifies, and vilifies also; but in the end it creates its own reality, its heterogeneous but usually coherent versions of events; and no sane human being ever trusts someone else’s version more than his own.”

What are the different ways in which ‘truth’ can be understood from the passage?

(A)  Truth is what can be verified by hard empirical evidence.

(B)  Truth is based on what can be perceived by the senses.

(C)  Truth is the product of memory that is fallible, selective and slanted.

(D)  Truth is contingent on the observer and can only be partial.

Answer: (C, D)

25. A firm needs both skilled labour and unskilled labour for the production of cloth. The wage of skilled labour is Rs. 40,000 per month, and that of unskilled labour is Rs. 15,000 per month. The total wage bill of the firm for the production of cloth is Rs. 23,75,000 in a month for 100 labour. How many skilled labour are employed by the firm (in Integer)?

Answer: (35 to 35)

26. Select the odd word and write the option number as answer:

(1) Lek (2) Zloty (3) Diner (4) Drachma (5) Real

Answer: (3 to 3)

Economics – C1

XH-C1: Q.27 – Q.44 Carry ONE mark Each

27. An individual is endowed with income of Rs. 142 and has the utility function U(x1, x2) = x2(x1 + 1), where x1 ≥ 0, x2 ≥ 0. The unit price of x1 is Rs. 2 and the unit price of x2 is Rs. 3. The utility maximizing bundle is

(A)  x1 = 35, x2 = 20

(B)  x1 = 30, x2 = 24

(C)  x1 = 35, x2 = 24

(D)  x1 = 30, x2 = 20

Answer: (C)

28. The International Monetary Fund (IMF) began operations in the year

(A)  1942

(B)  1947

(C)  1945

(D)  1940

Answer: (MTA)

29. According to the Working Group on Money Supply: Analytics and Methodology of Compilation (1998) constituted by the Reserve Bank of India (RBI), which of the following is NOT a component of the new monetary aggregate NM1 ?

(A)  Currency with the public

(B)  Demand deposits with the banking system

(C)  Short-term time deposits of residents

(D)  ‘Other’ deposits with the RBI

Answer: (C)

30. Stagflation is a situation when

(A)  both unemployment and inflation are low

(B)  both unemployment and inflation are high

(C)  unemployment is high but inflation is low

(D)  unemployment is low but inflation is high

Answer: (B)

31. Consider the Keynesian consumption function C = α + βY, where C is the aggregate consumption, Y is the aggregate income, α is a constant (α > 0), and β is the marginal propensity to consume (0 < β < 1). Then, the average propensity to consume is

(A)  α

(B) 

(C)  αY + βY2

(D) 

Answer: (B)

32. An analyst regressed Y on X1 and X2. If she later noticed that X1 = 5X2, then which of the following assumptions of the classical linear regression model was violated?

(A)  Homoscedasticity

(B)  No Perfect Multicollinearity

(C)  No Autocorrelation

(D)  Linearity in parameters

Answer: (B)

33. Which of the following is NOT an example of non-tariff barriers?

(A)  Voluntary export restraint

(B)  A procurement law directing a government to buy domestically made products unless comparable foreign made products are substantially cheaper.

(C)  Imposition of sanitary and phytosanitary measures on agricultural produce.

(D)  An antidumping law

Answer: (D)

34. Among the following, who first proposed that internal government debt does not create a burden for the future generation?

(A)  N. Gregory Mankiw

(B)  Martin Feldstein

(C)  Harvey S. Rosen

(D)  A. P. Lerner

Answer: (D)

35. Which of the following is an example of direct tax?

(A)  Sales tax

(B)  Customs duty

(C)  Individual income tax

(D)  Excise tax

Answer: (C)

36. In the context of endogenous growth theory, the Nobel laureate Paul Romer emphasized that “ideas” are

(A)  non-rival

(B)  rival with medium degree of excludability

(C)  rival with high degree of excludability

(D)  rival with low degree of excludability

Answer: (A)

37. In the Human Development Index (HDI), the longevity is measured by

(A)  child survival rate

(B)  healthy life expectancy

(C)  disability-adjusted life years

(D)  life expectancy at birth

Answer: (D)

38. Which of the following statements is correct about the Fourteenth Finance Commission?

(A)  The Commission was chaired by Dr. C. Rangarajan.

(B)  The Commission recommended achieving 90 percent metering of electricity by the end of the year 2012.

(C)  The Commission recommended an increase in the share of tax devolution to states to 42 percent of the divisible pool.

(D)  The Commission was mandated to make recommendations for the period 2010-2015.

Answer: (C)

39. Many scholars consider the study conducted by Dandekar and Rath in the 1960s as the first systematic assessment of poverty in independent India. Which option from the following is NOT correct about the study?

(A)  The study used the data on monthly per capita consumption expenditure (MPCE) from the 1960-61 round of the National Sample Surveys.

(B)  The study used the identical calorie norm for rural and urban areas.

(C)  The poverty head count ratio estimated by the study was higher for rural areas than that for urban areas.

(D)  The study used the same poverty line for all states.

Answer: (C)

40. Which of the following statements is/are correct about the Pradhan Mantri Kaushal Vikas Yojana (PMKVY)?

(A)  It has been a flagship scheme of the Ministry of Education.

(B)  It was launched in the year 2010.

(C)  The National Skill Development Corporation has been responsible for its implementation.

(D)  One of the objectives of PMKVY has been to enable a large number of Indian youth to take up industry-relevant skill training.

Answer: (C, D)

41. Which of the following is/are used for testing the assumption of normality?

(A)  Shapiro-Wilk test

(B)  Breusch-Godfrey test

(C)  Jarque-Bera test

(D)  Park test

Answer: (A, C)

42. Suppose Amar borrows Rs. 1000 from Ujala. After one year, Ujala wants Rs. 1100 back from Amar. The yield to maturity in percent (%) on this borrowing is _______ (round off to one decimal place).

Answer: (10.0 to 10.0)

43. A 250 ml bottle of mango juice costs USD 4 in the United States. If the exchange rate is 0.02 USD per Rupee, then the cost of the same bottle of mango juice in Rupees would be _______ (in integer).

Answer: (200 to 200)

44. The following table provides population information for different age groups in 2010 and 2017.

The percentage change in old-age dependency ratio from 2010 to 2017 is _______ (round off to two decimal places).

Answer: (15.00 to 17.00)

XH-C1: Q.45– Q.65 Carry TWO marks Each

45. A firm in a market with perfect competition has the following total cost (TC) function: TC(Q) = a + b(Q)

where Q is the quantity produced by the firm, 𝑎 is the fixed cost and b(Q) is the variable cost. What will happen if the fixed cost increases?

(A)  In the short-run, the firm’s Average Variable Cost (AVC) curve will shift upwards.

(B)  In the short-run, the firm’s Average Total Cost (ATC) curve will shift upwards.

(C)  The firm will earn higher profits.

(D)  In the short-run, the firm’s Marginal Cost (MC) curve will shift upwards.

Answer: (B)

46. The emission of greenhouse gases is an example of “bads” that are

(A)  rival and excludable

(B)  non-rival and excludable

(C)  rival and non-excludable

(D)  non-rival and non-excludable

Answer: (D)

47. Consider a closed-economy IS-LM model. The IS and LM equations are

where Y is the output, C is the consumption (C′ > 0), I is the investment (I′ < 0), z ≡ i – πe, i is the nominal interest rate, πe is the expected inflation,  is the government purchases,  is the fixed real money balances, and k and l are positive constants.

Suppose everyone in the economy suddenly expects the inflation to rise in the future. Assuming that the LM curve remains unchanged, what will happen in the short-run?

(A)  Equilibrium Y increases.

(B)  Aggregate demand remains unchanged.

(C)  Equilibrium Y remains unchanged.

(D)  Aggregate demand shifts down.

Answer: (A)

48. Consider the following simultaneous equations model:

Yt = β1 + β2Xt + β3Xt – 1 + β4Zt + μ1t        (1)

Zt = δ1 + δ2Yt + δ3Wt + μ2t                       (2)

Before estimating the above model, a researcher performed the test of identification using order and rank conditions, and found that equation (2) is over identified. Then, which of the following methods is appropriate to estimate equation (2) ?

(A)  Two-Stage Least Squares

(B)  Indirect Least Squares

(C)  Weighted Least Squares

(D)  Ordinary Least Squares

Answer: (A)

49. An income tax system is considered as progressive if the average tax rate rises with income. Consider an income tax schedule: T = p + t Y, where T denotes the tax liability, p is a constant, 𝑡 is the constant marginal tax rate, and Y is the income. For this tax schedule to be progressive, the value of p

(A)  must be positive

(B)  must be negative

(C)  must be zero

(D)  can be any value except zero

Answer: (B)

50. Match the following:

(A)  I → P ; II → Q ; III → R ; IV → S

(B)  I → P ; II → S ; III → Q ; IV → R

(C)  I → R ; II → S ; III → Q ; IV → P

(D)  I → S ; II → P ; III → R ; IV → Q

Answer: (B)

51. Consider two countries, India and Bangladesh, and two goods, Glass Bottle and Ceramic Plate, with labour requirements of production for a unit of each good given below:

Which of the following options is/are correct?

(A)  India has an absolute advantage in Glass Bottle production and a comparative disadvantage in Glass Bottle production.

(B)  India has an absolute advantage in Ceramic Plate production and a comparative disadvantage in Ceramic Plate production.

(C)  India has an absolute advantage in Ceramic Plate production and a comparative disadvantage in Glass Bottle production.

(D)  India has an absolute advantage in Glass Bottle production and a comparative disadvantage in Ceramic Plate production.

Answer: (A, C)

52. Suppose the own price elasticity of demand and income elasticity of demand are given by ep and eI, respectively. The subscript p represents own price of a good and the subscript I represents the income of the consumer. Identify the correct statement(s) from the following.

(A)  If 1 < ep < ∞, the demand is price inelastic.

(B)  Luxury goods are more price inelastic and the necessities are price elastic.

(C)  Luxury goods have eI > 1.

(D)  If 0 < ep < 1, the demand is price elastic.

Answer: (C)

53. Let πe be the expected inflation rate, 𝑖 be the nominal interest rate and r be the real interest rate. Which of the following statements is/are correct?

(A)  For small values of r and πe, r ≈ i – πe

(B)  When real interest rate is low, there are greater incentives to borrow and fewer incentives to lend.

(C)  Real interest rate reflects the real cost of borrowing.

(D)  If i = 8% and πe = 10%, then r is approximately (+) 2%.

Answer: (A, B, C)

54. Which of the following models explain(s) the upward-sloping aggregate supply curve in the short-run?

(A)  Sticky-wage model

(B)  Worker-misperception model

(C)  Imperfect-information model

(D)  Solow model

Answer: (A, B, C)

55. Consider a Mundell-Fleming model for a small open economy with perfect capital mobility. The goods market equation is

Y = C(Y) +  I(r*) + G + NX(e)

where 𝑌 is the output, 𝐶 is the consumption (C′ > 0), I is the investment (I′ < 0), G is the government purchases, and NX is the net exports (NX′ < 0), r∗ is the fixed world interest rate, and e is the exchange rate.

The money market equation is

where M is the money supply, is the fixed price level, and k and l are positive constants.

Which of the following policies is/are ineffective (i.e., have no impact on income) in the short-run?

(A)  Expansionary fiscal policy under floating exchange rate

(B)  Expansionary monetary policy under floating exchange rate

(C)  Expansionary fiscal policy under fixed exchange rate

(D)  Expansionary monetary policy under fixed exchange rate

Answer: (A, D)

56. In the context of Balance of Payments accounting, which of the following transactions is/are NOT recorded under the Current Account?

(A)  Merchandise trade

(B)  Unilateral transfer payments

(C)  Purchase of international financial assets

(D)  Purchase of foreign currency by the central bank

Answer: (C, D)

57. The demand and supply functions for a commodity are given by:

D(p) = 10 – 2p and S(p) = −2 + p

where D(p) and S(p) are the quantity demanded and supplied, respectively, and p (in USD) is the unit price of the good.

If the government sets a price ceiling of USD 3 per unit, then the increase in consumer surplus (in USD) is _______ (round off to two decimal places).

Answer: (0.71 to 0.79)

58. A duopoly faces the inverse market demand function p = 120 − Q, where p is the unit price (in Rs.) of the good being sold by firms A and B, and Q is the total output. Firm A has a constant marginal cost of Rs. 20, which is exactly half of firm B’s constant marginal cost. There is no fixed cost for both the firms. If there exists a Cournot-Nash equilibrium, Q is _______ (in integer).

Answer: (60 to 60)

59. Consider the following short-run cost function:

C(q) = 10q3 – 80q2 + 300q + 50

At the minimum average variable cost (AVC), the value of marginal cost (MC) is ______ (in integer).

Answer: (140 to 140)

60. Consider the Keynesian Cross Model with a linear consumption function and a zero tax, where the government purchase is Rs. 100 and the equilibrium income is Rs. 1300. If the government purchase is increased to Rs. 125, the equilibrium income increases to Rs. 1400. Using the given information, the marginal propensity to consume is _______ (round off to two decimal places).

Answer: (0.71 to 0.79)

61. Using the Ordinary Least Squares (OLS) method, a researcher estimated the relationship between initial salary (S) of MBA graduates and their cumulative grade point average (CGPA) as

where  The standard errors of  are 921.79 and 70.01, respectively.

The t-statistic for testing the null hypothesis β1 = 0 is ______ (round off to two decimal places).

Answer: (9.20 to 9.24)

62. Let X be a random variable with the probability density function f(x) such that

Then, the variance of X is _______ (in integer).

Answer: (1 to 1)

63. Suppose from the estimation of a linear regression model

Yi = β0 + β1Xi + ei

the residual sum of squares and the total sum of squares are obtained as 44 and 80, respectively. The value of coefficient of determination is _______ (round off to two decimal places).

Answer: (0.43 to 0.47)

64. A labour-augmenting production function is

Y = K0.33(AL)0.67

where Y = output, K = capital, L = labour, and A = technology.

Assume that the growth rate of L is 1.2 percent per annum, the growth rate of K is 3 percent per annum, and the growth rate of A is 1.5 percent per annum. Using the growth-accounting approach, the growth rate of Y in percent per annum is _______ (round off to two decimal places).

Answer: (2.70 to 2.90)

65. A monopolist is facing the demand function  where Q is the quantity demanded and P is the price per unit of the good (P > 1). The average variable cost for the monopolist is 4/√Q and the fixed cost is 10. The profit maximizing price is _______ (in integer).

Answer: (26 to 26)

XH : Humanities & Social Sciences

General Aptitude

Q.1 – Q.5 Carry ONE mark each.

1. Rafi told Mary, “I am thinking of watching a film this weekend.”

The following reports the above statement in indirect speech:

Rafi told Mary that he _______ of watching a film that weekend.

(A)  thought

(B)  is thinking

(C)  am thinking

(D)  was thinking

Answer: (D)

2. Permit : _______ : : Enforce : Relax

(By word meaning)

(A)  Allow

(B)  Forbid

(C)  License

(D)  Reinforce

Answer: (B)

3. Given a fair six-faced dice where the faces are labelled ‘1’, ‘2’, ‘3’, ‘4’, ‘5’, and ‘6’, what is the probability of getting a ‘1’ on the first roll of the dice and a ‘4’ on the second roll?

(A)  1/36

(B)  1/6

(C)  5/6

(D)  1/3

Answer: (A)

4. A recent survey shows that 65% of tobacco users were advised to stop consuming tobacco. The survey also shows that 3 out of 10 tobacco users attempted to stop using tobacco.

Based only on the information in the above passage, which one of the following options can be logically inferred with certainty?

(A)  A majority of tobacco users who were advised to stop consuming tobacco made an attempt to do so.

(B)  A majority of tobacco users who were advised to stop consuming tobacco did not attempt to do so.

(C)  Approximately 30% of tobacco users successfully stopped consuming tobacco.

(D)  Approximately 65% of tobacco users successfully stopped consuming tobacco.

Answer: (B)

5. How many triangles are present in the given figure?

(A)  12

(B)  16

(C)  20

(D)  24

Answer: (C)

Q.6 – Q.10 Carry TWO marks Each

6. Students of all the departments of a college who have successfully completed the registration process are eligible to vote in the upcoming college elections. However, by the time the due date for registration was over, it was found that suprisingly none of the students from the Department of Human Sciences had completed the registration process.

Based only on the information provided above, which one of the following sets of statement(s) can be logically inferred with certainty?

(i) All those students who would not be eligible to vote in the college elections would certainly belong to the Department of Human Sciences.

(ii) None of the students from departments other than Human Sciences failed to complete the registration process within the due time.

(iii) All the eligible voters would certainly be students who are not from the Department of Human Sciences.

(A)  (i) and (ii)

(B)  (i) and (iii)

(C)  only (i)

(D)  only (iii)

Answer: (D)

7. Which one of the following options represents the given graph?

(A)  f(x) = x22|x|

(B)  f(x) = x 2|x|

(C)  f(x) = |x|2x

(D)  f(x) = x2x

Answer: (B)

8. Which one of the options does NOT describe the passage below or follow from it?

We tend to think of cancer as a ‘modern’ illness because its metaphors are so modern. It is a disease of overproduction, of sudden growth, a growth that is unstoppable, tipped into the abyss of no control. Modern cell biology encourages us to imagine the cell as a molecular machine. Cancer is that machine unable to quench its intial command (to grow) and thus transform into an indestructible, self-propelled automaton.

[Adapted from The Emperor of All Maladies by Siddhartha Mukherjee]

(A)  It is a reflection of why cancer seems so modern to most of us.

(B)  It tells us that modern cell biology uses and promotes metaphors of machinery.

(C)  Modern cell biology encourages metaphors of machinery, and cancer is often imagined as a machine.

(D)  Modern cell biology never uses figurative language, such as metaphors, to describe or explain anything.

Answer: (D)

9. The digit in the unit’s place of the product 3999 × 71000 is ________.

(A)  7

(B)  1

(C)  3

(D)  9

Answer: (A)

10. A square with sides of length 6 cm is given. The boundary of the shaded region is defined by two semi-circles whose diameters are the sides of the square, as shown.

The area of the shaded region is _______ cm2.

(A)  6π

(B)  18

(C)  20

(D)  9π

Answer: (B)

Reasoning and Comprehension (XH-B1)

XH-B1: Q.11 – Q.17 Carry ONE mark Each

11. Which word below best describes the idea of being both Spineless and Cowardly?

(A)  Pusillanimous

(B)  Unctuous

(C)  Obsequious

(D)  Reticent

Answer: (A)

12. Choose the right preposition to fill up the blank:

The whole family got together ___ Diwali

(A)  of

(B)  at

(C)  in

(D)  till

Answer: (B)

13. Select the correct option to fill in all the blanks to complete the passage:

The (i) _______ factor amid this turbulence has been the (ii) ________ of high-octane, action-oriented films such as RRR, K.G.F: Chapter 2 and Pushpa from film industries in the south of the country. Traditionally, films made in the south have done well in their own (iii) _________. But increasingly, their dubbed versions have performed well in the Hindi heartland, with collections (iv) ________ those of their Bollywood counterparts.

(A)  (i) disheartening (ii) failure (iii) channels (iv) matching

(B)  (i) redeeming (ii) outperformance (iii) geographies (iv) eclipsing

(C)  (i) shocking (ii) underperformance (iii) cinemas (iv) below

(D)  (i) humbling (ii) bombing (iii) theatres (iv) falling behind

Answer: (B)

14. The following passage consists of 6 sentences. The first and sixth sentences of the passage are at their correct positions, while the middle four sentences (represented by 2, 3, 4, and 5) are jumbled up.

Choose the correct sequence of the sentences so that they form a coherent paragraph:

(1) Most obviously, mobility is taken to be a geographical as well as a social phenomenon.

(2) Much of the social mobility literature regarded society as a uniform surface and failed to register the geographical intersections of region, city and place, with the social categories of class, gender and ethnicity.

(3) The existing sociology of migration is incidentally far too limited in its concerns to be very useful here.

(4) Further, I am concerned with the flows of people within, but especially beyond, the territory of each society, and how these flows may relate to many different desires, for work, housing, leisure, religion, family relationships, criminal gain, asylum seeking and so on.

(5) Moreover, not only people are mobile but so too are many ‘objects’.

(6) I show that sociology’s recent development of a ‘sociology of objects’ needs to be taken further and that the diverse flows of objects across societal borders and their intersections with the multiple flows of people are hugely significant.

(A)  3, 2, 5, 4

(B)  2, 3, 4, 5

(C)  5, 4, 3, 2

(D)  4, 2, 5, 3

Answer: (B)

15. The population of a country increased by 5% from 2020 to 2021. Then, the population decreased by 5% from 2021 to 2022. By what percentage did the population change from 2020 to 2022?

(A)  -0.25%

(B)  0%

(C)  2.5%

(D)  10.25%

Answer: (A)

16. The words Thin: Slim: Slender are related in some way. Identify the correct option(s) that reflect(s) the same relationship:

(A)  Fat: Plump: Voluptuous

(B)  Short: Small: Petite

(C)  Tall: Taller: Tallest

(D)  Fair: Dark: Wheatish

Answer: (A, B)

17. A pandemic like situation hit the country last year, resulting in loss of human life and economic depression. To improve the condition of its citizens, the government made a series of emergency medical interventions and increased spending to revive the economy. In both these efforts, district administration authorities were actively involved.

Which of the following action(s) are plausible?

(A)  In future, the government can make district administration authorities responsible for protecting health of citizens and reviving the economy.

(B)  The government may set up a task force to review the post pandemic situation and ascertain the effectiveness of the measures taken.

(C)  The government may set up a committee to formulate a pandemic management program to minimize losses to life and economy in future.

(D)  The government may take population control measures to minimize pandemic related losses in future.

Answer: (B, C)

XH-B1: Q.18 – Q.26 Carry TWO marks Each

18. Six students, Arif, Balwinder, Chintu, David, Emon and Fulmoni appeared in the GATE-XH exam in 2022. Balwinder scores less than Chintu in XH-B1, but more than Arif in XH-C1. David scores more than Balwinder in XH-C1, and more than Chintu in XH-B1. Emon scores less than David, but more than Fulmoni in XH-B1. Fulmoni scores more than David in XH-C1. Arif scores less than Emon, but more than Fulmoni in XH-B1. Who scores highest in XH-B1?

(A)  Fulmoni

(B)  Emon

(C)  David

(D)  Chintu

Answer: (C)

19. Select the correct relation between E and F.

(A)  E > F

(B)  E < F

(C)  E = F

(D)  E < -F

Answer: (B)

20. A code language is formulated thus:

Vowels in the original word are replaced by the next vowel from the list of vowels, A-E-I-O-U (For example, E is replaced by I and U is replaced by A). Consonants in the original word are replaced by previous consonant (For example, T is replaced by S and V is replaced by T).

Then how does the word, GOODMORNING appear in the coded language?

(A)  HUUFNUSPOPH

(B)  FIICLIQMEMF

(C)  FUUCLUQMOMF

(D)  HEEDATTACRH

Answer: (C)

21. The stranger is by nature no “owner of soil” — soil not only in the physical, but also in the figurative sense of a life-substance, which is fixed, if not in a point in space, at least in an ideal point of the social environment. Although in more intimate relations, he may develop all kinds of charm and significance, as long as he is considered a stranger in the eyes of the other, he is not an “owner of soil.” Restriction to intermediary trade, and often (as though sublimated from it) to pure finance, gives him the specific character of mobility. If mobility takes place within a closed group, it embodies that synthesis of nearness and distance which constitutes the formal position of the stranger. For, the fundamentally mobile person comes in contact, at one time or another, with every individual, but is not organically connected, through established ties of kinship, locality, and occupation, with any single one.

What assumptions can be made about the stranger from the passage above?

(A)  The stranger can become an owner of soil through developing all kinds of charm in more intimate relations.

(B)  The stranger cannot become an owner of soil either in the physical or psychological sense.

(C)  The stranger can become an owner of soil through establishing ties of kinship and so on.

(D)  The stranger might become an owner of soil in the physical sense but not in the psychological

Answer: (B)

22. L is the only son of A and S. S has one sibling, B, who is married to L’s aunt, K. B is the only son of D. How are L and D related?

Select the possible option(s):

(A)  Grandchild and Paternal Grandfather

(B)  Grandchild and Maternal Grandfather

(C)  Grandchild and Paternal Grandmother

(D)  Grandchild and Maternal Grandmother

Answer: (B, D)

23. Five segments of a sentence are given below. The first and fifth segments are at their correct positions, while the middle three segments (represented by 2, 3, and 4) are jumbled up. Choose the correct order of the segments so that they form a coherent sentence:

(1) Consumed multitudes are jostling and shoving inside me

(2) and guided only by the memory of a large white bedsheet with a roughly circular hole some seven inches in diameter cut into the center,

(3) clutching at the dream of that holey, mutilated square of linen, which is my talisman, my open-sesame,

(4) I must commence the business of remaking my life from the point at which it really began,

(5) some thirty-two years before anything as obvious, as present, as my clock-ridden, crime-stained birth.

(A)  2 – 3 – 4

(B)  3 – 2– 4

(C)  4 – 2– 3

(D)  4 – 3 – 2

Answer: (A)

24. “I told you the truth,” I say yet again, “Memory’s truth, because memory has its own special kind. It selects, eliminates, alters, exaggerates, minimizes, glorifies, and vilifies also; but in the end it creates its own reality, its heterogeneous but usually coherent versions of events; and no sane human being ever trusts someone else’s version more than his own.”

What are the different ways in which ‘truth’ can be understood from the passage?

(A)  Truth is what can be verified by hard empirical evidence.

(B)  Truth is based on what can be perceived by the senses.

(C)  Truth is the product of memory that is fallible, selective and slanted.

(D)  Truth is contingent on the observer and can only be partial.

Answer: (C, D)

25. A firm needs both skilled labour and unskilled labour for the production of cloth. The wage of skilled labour is Rs. 40,000 per month, and that of unskilled labour is Rs. 15,000 per month. The total wage bill of the firm for the production of cloth is Rs. 23,75,000 in a month for 100 labour. How many skilled labour are employed by the firm (in Integer)?

Answer: (35 to 35)

26. Select the odd word and write the option number as answer:

(1) Lek (2) Zloty (3) Diner (4) Drachma (5) Real

Answer: (3 to 3)

English – C2

XH-C2: Q.27 – Q.44 Carry ONE mark Each

27. Who published the novel The Bell Jar under the pseudonym Victoria Lucas?

(A)  Dorothy Richardson

(B)  Virginia Woolf

(C)  Sylvia Plath

(D)  Alice Walker

Answer: (C)

28. In which collection did Walt Whitman’s poem “Song of Myself” first appear?

(A)  Two Rivulets

(B)  November Boughs

(C)  The Golden Bough

(D)  Leaves of Grass

Answer: (D)

29. Who wrote the introduction to Rabindranath Tagore’s Gitanjali?

(A)  T. S. Eliot

(B)  Ezra Pound

(C)  W. H. Auden

(D)  W. B. Yeats

Answer: (D)

30. Identify the title of the poem in which the following lines appear:

“He was found by the Bureau of Statistics to be

One against whom there was no official complaint,

And all the reports on his conduct agree

That, in the modern sense of an old-fashioned word, he was a saint

For in everything he did he served the greater community.”

(A)  “In Memory of W. B. Yeats”

(B)  “The Unknown Citizen”

(C)  “In Praise of Limestone”

(D)  “On this Island”

Answer: (B)

31. Identify the point of view used in the following passage:

“You are not the kind of guy who would be at a place like this at this time of the morning. But here you are, and you cannot say that the terrain is entirely unfamiliar, though the details are fuzzy.”

(A)  Third-person point of view

(B)  The limited point of view

(C)  Second-person point of view

(D)  First-person point of view

Answer: (C)

32. Which of the following is a novel by Charles Dickens?

(A)  The Old Curiosity Shop

(B)  The Old Wives’ Tale

(C)  The Old Bachelor

(D)  One Hundred Years of Solitude

Answer: (A)

33. Which linguistic process can be seen in the formation of the following words?

(i) smog, (ii) brunch, (iii) motel (iv) telecast

(A)  Borrowing

(B)  Compounding

(C)  Blending

(D)  Backformation

Answer: (C)

34. Which writer is credited with the ‘chutneyfication’ of Indian English?

(A)  Raja Rao

(B)  Salman Rushdie

(C)  Amitav Ghosh

(D)  Arundhati Roy

Answer: (B)

35. Whom would you associate the term ‘simulacra’ with?

(A)  Noam Chomsky

(B)  Jean Baudrillard

(C)  Felix Guattari

(D)  Michel Foucault

Answer: (B)

36. In Plato’s idea of the Republic there is no place for the ___________.

(A)  Lawyer

(B)  Magistrate

(C)  Politician

(D)  Poet

Answer: (D)

37. What was Aristotle’s definition of hubris?

(A)  Tragic flaw in a character

(B)  A false sense of pride which eventually causes the character’s downfall

(C)  An ability to imagine the future

(D)  A humble, ascetic quality

Answer: (B)

38. Stephen Dedalus is a recurring character in the works of _________.

(A)  James Joyce

(B)  H. G. Wells

(C)  P. G. Wodehouse

(D)  D. H. Lawrence

Answer: (A)

39. Which of the following novels opens with the sentence, “It is a truth universally acknowledged that a single man in possession of a good fortune must be in want of a wife.”?

(A)  Sense and Sensibility

(B)  Pride and Prejudice

(C)  Mansfield Park

(D)  Emma

Answer: (B)

40. Which of the following novels by Chinua Achebe derives its title from W. B. Yeats’s poem “The Second Coming”?

(A)  Arrow of God

(B)  No Longer at Ease

(C)  A Man of the People

(D)  Things Fall Apart

Answer: (D)

41. Identify the novels that deal with the trauma of Partition:

(A)  Shauna Singh Baldwin’s What the Body Remembers

(B)  Amitav Ghosh’s The Shadow Lines

(C)  Anita Desai’s Cry, the Peacock

(D)  Kamala Markandaya’s Nectar in a Sieve

Answer: (A, B)

42. Which of the following writers are associated with the Theatre of the Absurd?

(A)  Harold Pinter

(B)  Edward Albee

(C)  John Osborne

(D)  Eugene O’Neill

Answer: (A, B)

43. Identify the writers who are referred to as ‘metaphysical poets’:

(A)  John Donne

(B)  Andrew Marvell

(C)  Philip Larkin

(D)  T. S. Eliot

Answer: (A, B)

44. What are the sources of Girish Karnad’s Hayavadana?

(A)  Thomas Mann’s The Transported Heads

(B)  Valmiki’s Ramayana

(C)  Somadeva’s Kathasaritsagara

(D)  Franz Kafka’s The Metamorphosis

Answer: (A, C)

XH-C2: Q.45– Q.65 Carry TWO marks Each

45. Which of the following terms are used by Samuel Taylor Coleridge in his theory of imagination?

(A)  Primary imagination, secondary imagination, and fancy

(B)  Negative capability, Hellenism, and impersonality

(C)  Egotistical sublime, oversoul, and pantheism

(D)  Unacknowledged legislation, atheism, and anarchy

Answer: (A)

46. Which of the following is the forerunner of the autobiography?

(A)  St. Augustine’s Confessions

(B)  James Joyce’s Portrait of the Artist as a Young Man

(C)  William Wordsworth’s The Prelude

(D)  Izaak Walton’s Lives

Answer: (A)

47. Which of the following poems did Robert Browning intend to write as a play?

(A)  “Men and Women”

(B)  “Dramatis Personae”

(C)  “The Inn Album”

(D)  “The Ring and the Book”

Answer: (C)

48. The ‘Age of Reason’ in English literary history is popularly known as:

(A)  The Medieval Period

(B)  The Neo-classical Age

(C)  The Romantic Age

(D)  The Victorian Age

Answer: (B)

49. Who first translated Jacques Derrida’s work into English?

(A)  Gayatri C. Spivak

(B)  Edward Said

(C)  Harold Bloom

(D)  Paul de Man

Answer: (A)

50. Identify the ‘Lake Poets’:

(A)  Byron, Shelley, Keats

(B)  Wordsworth, Coleridge, Byron

(C)  Byron, Southey, Wordsworth

(D)  Wordsworth, Coleridge, Southey

Answer: (D)

51. Choose from the following options the type of drama that is intended by the author to be read rather than to be performed:

(A)  Kitchen Sink Drama

(B)  Closet Drama

(C)  Poetic Drama

(D)  Folk Drama

Answer: (B)

52. Identify the commonality shared by the authors of Mansfield Park and Middle March:

(A)  Both the novels were authored by men who were sent on exile.

(B)  Both the novels were authored by political prisoners.

(C)  Both the novels were written by children who were not allowed to publish their works.

(D)  Both the novels were written by women who wrote under pseudonyms.

Answer: (D)

53. Who said, “Poetry makes nothing happen”?

(A)  Marianne Moore

(B)  Ezra Pound

(C)  Wallace Stevens

(D)  W. H. Auden

Answer: (D)

54. Which literary device does the following line employ?

“A timorous foe, and a suspicious friend.”

(A)  Antithesis

(B)  Antistrophe

(C)  Oxymoron

(D)  Apostrophe

Answer: (A)

55. Match the following excerpts with their authors:

(A)  (P)-(iii), (Q)-(iv), (R)-(i), (S)-(ii)

(B)  (P)-(iv), (Q)-(iii), (R)-(ii), (S)-(i)

(C)  (P)-(iii), (Q)-(ii), (R)-(i), (S)-(iv)

(D)  (P)-(i), (Q)-(iv), (R)-(ii), (S)-(iii)

Answer: (A)

56. The 1667 edition of Paradise Lost had 10 books. How many more were added to the 1674 edition?

(A)  2

(B)  4

(C)  6

(D)  12

Answer: (A)

57. Read the following poem and identify the appropriate options:

And search

for certain thin –

stemmed, bubble-eyed water bugs.

See them perch

on dry capillary legs

weightless

on the ripple skin

of a stream.

No, not only prophets

walk on water. This bug sits

on a landslide of lights

and drowns eye –

deep

into its tiny strip

of sky.

(A)  It uses free verse form.

(B)  It employs imagery.

(C)  It uses the iambic pentameter.

(D)  It juxtaposes the non-human with the human.

Answer: (A, B, D)

58. Which of the following employ ‘Interior Monologue’?

(A)  Alfred Lord Tennyson’s “In Memoriam”

(B)  The final chapter of James Joyce’s Ulysses

(C)  T. S. Eliot’s “The Love Song of J. Alfred Prufrock”

(D)  Robert Browning’s “My Last Duchess”

Answer: (B, C)

59. Which of the following works may be described as novels in verse?

(A)  Aurora Leigh by Elizabeth Barrett Browning

(B)  The Golden Gate by Vikram Seth

(C)  Pamela by Samuel Richardson

(D)  Old Possum’s Book of Practical Cats by T. S. Eliot

Answer: (A, B)

60. Which of the following critics belong to the deconstructionist school?

(A)  Jacques Derrida

(B)  Paul de Man

(C)  J. Hillis Miller

(D)  Kate Soper

Answer: (A, B, C)

61. Cleanth Brooks’s definition of ‘paradox’ in poetry foregrounds the following qualities:

(A)  Wonder and irony

(B)  Contradiction and qualification

(C)  Piety and plurality

(D)  Omniscience and death of the author

Answer: (A, B)

62. Two examples of magic realist fiction include:

(A)  Midnight’s Children

(B)  The Tin Drum

(C)  The English Teacher

(D)  Tom Jones

Answer: (A, B)

63. Ferdinand de Saussure differentiates language in terms of:

(A)  langue

(B)  metaphor

(C)  metonymy

(D)  parole

Answer: (A, D)

64. Which of the following are considered to be typical postmodern narratives?

(A)  Italo Calvino’s If on a Winter’s Night a Traveller

(B)  John Barth’s Lost in the Funhouse

(C)  Thomas Pynchon’s V.

(D)  Iris Murdoch’s The Bell

Answer: (A, B, C)

65. What does a green reading of a text aim at?

(A)  Analyzing the implications of a text for environmental concerns

(B)  Deconstructing human exceptionalism

(C)  Studying connections between humans, society and the non-human world

(D)  Marginalizing differently abled people

Answer: (A, B, C)

XH : Humanities & Social Sciences

General Aptitude

Q.1 – Q.5 Carry ONE mark each.

1. Rafi told Mary, “I am thinking of watching a film this weekend.”

The following reports the above statement in indirect speech:

Rafi told Mary that he _______ of watching a film that weekend.

(A)  thought

(B)  is thinking

(C)  am thinking

(D)  was thinking

Answer: (D)

2. Permit : _______ : : Enforce : Relax

(By word meaning)

(A)  Allow

(B)  Forbid

(C)  License

(D)  Reinforce

Answer: (B)

3. Given a fair six-faced dice where the faces are labelled ‘1’, ‘2’, ‘3’, ‘4’, ‘5’, and ‘6’, what is the probability of getting a ‘1’ on the first roll of the dice and a ‘4’ on the second roll?

(A)  1/36

(B)  1/6

(C)  5/6

(D)  1/3

Answer: (A)

4. A recent survey shows that 65% of tobacco users were advised to stop consuming tobacco. The survey also shows that 3 out of 10 tobacco users attempted to stop using tobacco.

Based only on the information in the above passage, which one of the following options can be logically inferred with certainty?

(A)  A majority of tobacco users who were advised to stop consuming tobacco made an attempt to do so.

(B)  A majority of tobacco users who were advised to stop consuming tobacco did not attempt to do so.

(C)  Approximately 30% of tobacco users successfully stopped consuming tobacco.

(D)  Approximately 65% of tobacco users successfully stopped consuming tobacco.

Answer: (B)

5. How many triangles are present in the given figure?

(A)  12

(B)  16

(C)  20

(D)  24

Answer: (C)

Q.6 – Q.10 Carry TWO marks Each

6. Students of all the departments of a college who have successfully completed the registration process are eligible to vote in the upcoming college elections. However, by the time the due date for registration was over, it was found that suprisingly none of the students from the Department of Human Sciences had completed the registration process.

Based only on the information provided above, which one of the following sets of statement(s) can be logically inferred with certainty?

(i) All those students who would not be eligible to vote in the college elections would certainly belong to the Department of Human Sciences.

(ii) None of the students from departments other than Human Sciences failed to complete the registration process within the due time.

(iii) All the eligible voters would certainly be students who are not from the Department of Human Sciences.

(A)  (i) and (ii)

(B)  (i) and (iii)

(C)  only (i)

(D)  only (iii)

Answer: (D)

7. Which one of the following options represents the given graph?

(A)  f(x) = x22|x|

(B)  f(x) = x 2|x|

(C)  f(x) = |x|2x

(D)  f(x) = x2x

Answer: (B)

8. Which one of the options does NOT describe the passage below or follow from it?

We tend to think of cancer as a ‘modern’ illness because its metaphors are so modern. It is a disease of overproduction, of sudden growth, a growth that is unstoppable, tipped into the abyss of no control. Modern cell biology encourages us to imagine the cell as a molecular machine. Cancer is that machine unable to quench its intial command (to grow) and thus transform into an indestructible, self-propelled automaton.

[Adapted from The Emperor of All Maladies by Siddhartha Mukherjee]

(A)  It is a reflection of why cancer seems so modern to most of us.

(B)  It tells us that modern cell biology uses and promotes metaphors of machinery.

(C)  Modern cell biology encourages metaphors of machinery, and cancer is often imagined as a machine.

(D)  Modern cell biology never uses figurative language, such as metaphors, to describe or explain anything.

Answer: (D)

9. The digit in the unit’s place of the product 3999 × 71000 is ________.

(A)  7

(B)  1

(C)  3

(D)  9

Answer: (A)

10. A square with sides of length 6 cm is given. The boundary of the shaded region is defined by two semi-circles whose diameters are the sides of the square, as shown.

The area of the shaded region is _______ cm2.

(A)  6π

(B)  18

(C)  20

(D)  9π

Answer: (B)

Reasoning and Comprehension (XH-B1)

XH-B1: Q.11 – Q.17 Carry ONE mark Each

11. Which word below best describes the idea of being both Spineless and Cowardly?

(A)  Pusillanimous

(B)  Unctuous

(C)  Obsequious

(D)  Reticent

Answer: (A)

12. Choose the right preposition to fill up the blank:

The whole family got together ___ Diwali

(A)  of

(B)  at

(C)  in

(D)  till

Answer: (B)

13. Select the correct option to fill in all the blanks to complete the passage:

The (i) _______ factor amid this turbulence has been the (ii) ________ of high-octane, action-oriented films such as RRR, K.G.F: Chapter 2 and Pushpa from film industries in the south of the country. Traditionally, films made in the south have done well in their own (iii) _________. But increasingly, their dubbed versions have performed well in the Hindi heartland, with collections (iv) ________ those of their Bollywood counterparts.

(A)  (i) disheartening (ii) failure (iii) channels (iv) matching

(B)  (i) redeeming (ii) outperformance (iii) geographies (iv) eclipsing

(C)  (i) shocking (ii) underperformance (iii) cinemas (iv) below

(D)  (i) humbling (ii) bombing (iii) theatres (iv) falling behind

Answer: (B)

14. The following passage consists of 6 sentences. The first and sixth sentences of the passage are at their correct positions, while the middle four sentences (represented by 2, 3, 4, and 5) are jumbled up.

Choose the correct sequence of the sentences so that they form a coherent paragraph:

(1) Most obviously, mobility is taken to be a geographical as well as a social phenomenon.

(2) Much of the social mobility literature regarded society as a uniform surface and failed to register the geographical intersections of region, city and place, with the social categories of class, gender and ethnicity.

(3) The existing sociology of migration is incidentally far too limited in its concerns to be very useful here.

(4) Further, I am concerned with the flows of people within, but especially beyond, the territory of each society, and how these flows may relate to many different desires, for work, housing, leisure, religion, family relationships, criminal gain, asylum seeking and so on.

(5) Moreover, not only people are mobile but so too are many ‘objects’.

(6) I show that sociology’s recent development of a ‘sociology of objects’ needs to be taken further and that the diverse flows of objects across societal borders and their intersections with the multiple flows of people are hugely significant.

(A)  3, 2, 5, 4

(B)  2, 3, 4, 5

(C)  5, 4, 3, 2

(D)  4, 2, 5, 3

Answer: (B)

15. The population of a country increased by 5% from 2020 to 2021. Then, the population decreased by 5% from 2021 to 2022. By what percentage did the population change from 2020 to 2022?

(A)  -0.25%

(B)  0%

(C)  2.5%

(D)  10.25%

Answer: (A)

16. The words Thin: Slim: Slender are related in some way. Identify the correct option(s) that reflect(s) the same relationship:

(A)  Fat: Plump: Voluptuous

(B)  Short: Small: Petite

(C)  Tall: Taller: Tallest

(D)  Fair: Dark: Wheatish

Answer: (A, B)

17. A pandemic like situation hit the country last year, resulting in loss of human life and economic depression. To improve the condition of its citizens, the government made a series of emergency medical interventions and increased spending to revive the economy. In both these efforts, district administration authorities were actively involved.

Which of the following action(s) are plausible?

(A)  In future, the government can make district administration authorities responsible for protecting health of citizens and reviving the economy.

(B)  The government may set up a task force to review the post pandemic situation and ascertain the effectiveness of the measures taken.

(C)  The government may set up a committee to formulate a pandemic management program to minimize losses to life and economy in future.

(D)  The government may take population control measures to minimize pandemic related losses in future.

Answer: (B, C)

XH-B1: Q.18 – Q.26 Carry TWO marks Each

18. Six students, Arif, Balwinder, Chintu, David, Emon and Fulmoni appeared in the GATE-XH exam in 2022. Balwinder scores less than Chintu in XH-B1, but more than Arif in XH-C1. David scores more than Balwinder in XH-C1, and more than Chintu in XH-B1. Emon scores less than David, but more than Fulmoni in XH-B1. Fulmoni scores more than David in XH-C1. Arif scores less than Emon, but more than Fulmoni in XH-B1. Who scores highest in XH-B1?

(A)  Fulmoni

(B)  Emon

(C)  David

(D)  Chintu

Answer: (C)

19. Select the correct relation between E and F.

(A)  E > F

(B)  E < F

(C)  E = F

(D)  E < -F

Answer: (B)

20. A code language is formulated thus:

Vowels in the original word are replaced by the next vowel from the list of vowels, A-E-I-O-U (For example, E is replaced by I and U is replaced by A). Consonants in the original word are replaced by previous consonant (For example, T is replaced by S and V is replaced by T).

Then how does the word, GOODMORNING appear in the coded language?

(A)  HUUFNUSPOPH

(B)  FIICLIQMEMF

(C)  FUUCLUQMOMF

(D)  HEEDATTACRH

Answer: (C)

21. The stranger is by nature no “owner of soil” — soil not only in the physical, but also in the figurative sense of a life-substance, which is fixed, if not in a point in space, at least in an ideal point of the social environment. Although in more intimate relations, he may develop all kinds of charm and significance, as long as he is considered a stranger in the eyes of the other, he is not an “owner of soil.” Restriction to intermediary trade, and often (as though sublimated from it) to pure finance, gives him the specific character of mobility. If mobility takes place within a closed group, it embodies that synthesis of nearness and distance which constitutes the formal position of the stranger. For, the fundamentally mobile person comes in contact, at one time or another, with every individual, but is not organically connected, through established ties of kinship, locality, and occupation, with any single one.

What assumptions can be made about the stranger from the passage above?

(A)  The stranger can become an owner of soil through developing all kinds of charm in more intimate relations.

(B)  The stranger cannot become an owner of soil either in the physical or psychological sense.

(C)  The stranger can become an owner of soil through establishing ties of kinship and so on.

(D)  The stranger might become an owner of soil in the physical sense but not in the psychological

Answer: (B)

22. L is the only son of A and S. S has one sibling, B, who is married to L’s aunt, K. B is the only son of D. How are L and D related?

Select the possible option(s):

(A)  Grandchild and Paternal Grandfather

(B)  Grandchild and Maternal Grandfather

(C)  Grandchild and Paternal Grandmother

(D)  Grandchild and Maternal Grandmother

Answer: (B, D)

23. Five segments of a sentence are given below. The first and fifth segments are at their correct positions, while the middle three segments (represented by 2, 3, and 4) are jumbled up. Choose the correct order of the segments so that they form a coherent sentence:

(1) Consumed multitudes are jostling and shoving inside me

(2) and guided only by the memory of a large white bedsheet with a roughly circular hole some seven inches in diameter cut into the center,

(3) clutching at the dream of that holey, mutilated square of linen, which is my talisman, my open-sesame,

(4) I must commence the business of remaking my life from the point at which it really began,

(5) some thirty-two years before anything as obvious, as present, as my clock-ridden, crime-stained birth.

(A)  2 – 3 – 4

(B)  3 – 2– 4

(C)  4 – 2– 3

(D)  4 – 3 – 2

Answer: (A)

24. “I told you the truth,” I say yet again, “Memory’s truth, because memory has its own special kind. It selects, eliminates, alters, exaggerates, minimizes, glorifies, and vilifies also; but in the end it creates its own reality, its heterogeneous but usually coherent versions of events; and no sane human being ever trusts someone else’s version more than his own.”

What are the different ways in which ‘truth’ can be understood from the passage?

(A)  Truth is what can be verified by hard empirical evidence.

(B)  Truth is based on what can be perceived by the senses.

(C)  Truth is the product of memory that is fallible, selective and slanted.

(D)  Truth is contingent on the observer and can only be partial.

Answer: (C, D)

25. A firm needs both skilled labour and unskilled labour for the production of cloth. The wage of skilled labour is Rs. 40,000 per month, and that of unskilled labour is Rs. 15,000 per month. The total wage bill of the firm for the production of cloth is Rs. 23,75,000 in a month for 100 labour. How many skilled labour are employed by the firm (in Integer)?

Answer: (35 to 35)

26. Select the odd word and write the option number as answer:

(1) Lek (2) Zloty (3) Diner (4) Drachma (5) Real

Answer: (3 to 3)

Linguistics – C3

XH-C3: Q.27 – Q.44 Carry ONE mark Each

27. In a writing system, if each unique grapheme represents a unique morpheme/word, the writing system is known as _____________.

(A)  Logographic

(B)  Syllabic

(C)  Abugida

(D)  Moraic

Answer: (A)

28. If an SOV language allows movement to OSV and VSO word orders, the resulting word order will be known as:

(A)  Unmarked

(B)  Marked

(C)  Ungrammatical

(D)  Default

Answer: (B)

29. The phenomenon where ‘missed hotel’ is pronounced as ‘hissed motel’ is known as _____________.

(A)  Agrammatism

(B)  Wernicke’s aphasia

(C)  Spoonerism

(D)  Malapropism

Answer: (C)

30. When new information is introduced in a communication, such information is classified as __________.

(A)  Focus

(B)  Topic

(C)  Presupposition

(D)  Theme

Answer: (A)

31. What type of morphological process is involved in the expression ‘English-vinglish’

(A)  Complete reduplication

(B)  Partial reduplication

(C)  Prefixation

(D)  Suffixation

Answer: (B)

32. Which one of the following is a language isolate?

(A)  Burushashki

(B)  Mundari

(C)  Angami

(D)  Kalasha

Answer: (A)

33. Which of the following Dravidian language is spoken in Pakistan?

(A)  Konda

(B)  Kuvi

(C)  Toda

(D)  Brahui

Answer: (D)

34. Which word-order pair is the most common in world’s languages?

(A)  SVO-SOV

(B)  SOV-VSO

(C)  VSO-VOS

(D)  SOV-VOS

Answer: (A)

35. Which type of morphological process is involved in creating catty from cat?

(A)  Inflection

(B)  Derivation

(C)  Suppletion

(D)  Reduplication

Answer: (B)

36. Devanagari organizes the consonant graphemes as shown in the image. What is the parameter by which the following two rows differ?

(A)  Voicing

(B)  Manner of articulation

(C)  Place of articulation

(D)  Aspiration

Answer: (C)

37. Which option is related to Neo-Whorfism?

(A)  Language, thought, worldview

(B)  Innateness, deep-structure, surface-structure

(C)  Language, methods, analysis

(D)  Signifier, signified, sign

Answer: (A)

38. Which of the following is/are involved in the comparative method for establishing language families and genetic relationship among languages?

(A)  Reconstruction of the proto language

(B)  Assembling a list of cognates

(C)  Strictly using basic vocabulary lists

(D)  Setting up sound correspondences

Answer: (A, B, D)

39. Aphasia can be caused by

(A)  Open or closed head trauma

(B)  Attention deficit

(C)  Neurodegeneration in advanced age

(D)  Lack of motivation

Answer: (A, C)

40. In sociolinguistics, the term ‘anti-language’ refers to the language used for

(A)  Academic purposes by professional criminologists

(B)  Legal proceedings in the court of law

(C)  Communication by small non-mainstream groups

(D)  Communication between caregivers and infants

Answer: (C)

41. In language policy making, which of the following steps is/are necessary for an erstwhile minority language, spoken by a sizeable population, to be introduced as a medium of instruction in schools?

(A)  Language revival

(B)  Corpus planning

(C)  Status planning

(D)  Language preservation

Answer: (B, C)

42. Which of the following research methods involve Reaction Time?

(A)  Behavioral methods

(B)  Experimental methods

(C)  Non-behavioral methods

(D)  Qualitative methods

Answer: (A, B)

43. If [p] and [ph] are allophones of the same phoneme /p/, which of the following statements is/are true?

(A)  [p] and [ph] are in contrastive distribution

(B)  [p] and [ph] are in complementary distribution

(C)  [ph] has more restrictive occurrence than [p]

(D)  [p] and [ph] can be used interchangeably

Answer: (B, C)

44. Which of the following show(s) dissociation between cognitive disorder and language abilities?

(A)  Autism spectrum disorder

(B)  Williams syndrome

(C)  Dyslexia

(D)  Specific Language Impairment (SLI)

Answer: (B, C, D)

XH-C3: Q.45 – Q.65 Carry TWO mark Each

45. Which of the following sound pairs differ from each other in exactly two articulatory parameters?

(A)  [p] vs. [b]

(B)  [t] vs. [s]

(C)  [v] vs. [θ]

(D)  [n] vs. [d]

Answer: (C)

46. The cover term determiner refers to:

(A)  Articles, demonstratives, and possessors

(B)  Possessors, prepositions, and demonstratives

(C)  Postpositions, articles, and prepositions

(D)  Articles, prepositions, and possessors

Answer: (A)

47. In ‘John seems to have left’, the subject John has undergone:

(A)  Subject-to-Subject lowering

(B)  Object-to-Subject raising

(C)  Subject-to-Subject raising

(D)  Object-to-Object lowering

Answer: (C)

48. The comparative and superlative forms of the adjective ‘good’ are examples of:

(A)  Alternation

(B)  Syncope

(C)  Ablaut

(D)  Suppletion

Answer: (D)

49. Consider the following question and determine where was is moved finally, in accordance with the minimalist assumptions:

Was John, who wrote the book, angry?

(A)  T to C

(B)  C to T

(C)  V to T

(D)  T to V

Answer: (A)

50. Match the following historical sound changes in Column X with the processes in Column Y

(A)  P-2, Q-4, R-1, S-3

(B)  P-4, Q-2, R-1, S-3

(C)  P-2, Q-1, R-4, S-3

(D)  P-2, Q-4, R-3, S-1

Answer: (A)

51. Consider the following morphological break-up of pfeifing produced by a simultaneous bilingual child. What phenomenon does this example indicate?

(A)  Code-mixing arising out of social bilingualism

(B)  Code-switching based on context of the conversation

(C)  Understanding both languages as part of a single ‘system’

(D)  Mixed language arising out of pedagogical preferences

Answer: (C)

52. Which one of the following is compounding of compounded words?

(A)  Lighthouse tower

(B)  Skating board

(C)  Boyfriend

(D)  Walkman

Answer: (A)

53. What sociolinguistic phenomenon does the following sentence exemplify?

(A)  Code switching

(B)  Bilingualism

(C)  Pidgin

(D)  Mixed language

Answer: (D)

54. Identify the labels for X and Y in the following tree.

(A)  X: Verb, Y: Adjective

(B)  X: Adjective, Y: Adjective

(C)  X: Adjective, Y: Verb

(D)  X: Verb, Y: Verb

Answer: (A)

55. From the following, which is the correct order of animacy hierarchy?

(A)  1st/2nd Person > 3rd Person > Proper Name > Human Noun

(B)  3rd Person > 1st/2nd Person > Proper Name > Human Noun

(C)  3rd Person > 1st/2nd Person > Human Noun > Proper Name

(D)  1st/2nd Person > 3rd Person > Human Noun > Proper Name

Answer: (A)

56. Look at the adjectives in Column X and match with the types of adjectives in Column Y. Choose the correct option.

(A)  P-2, Q-3, R-1

(B)  P-2, Q-1, R-3

(C)  P-1, Q-2, R-3

(D)  P-1, Q-3, R-2

Answer: (A)

57. In a given constraint ranking of *COMPLEX, DEP-IO >> MAX-IO for the input /spun/, which of the following candidates is/are optimal?

(A)  [su.pun]

(B)  [pun]

(C)  [sun]

(D)  [is.pun]

Answer: (B, C)

58. Consider the pattern of stress assignment shown below. Which of the given statements is/are true?

(A)  Stress assignment is from left to right

(B)  There are/is extrametrical syllable(s)

(C)  Feet are binary

(D)  Stress system is iambic

Answer: (A, B, C)

59. In word processing studies, visual word recognition is affected by:

(A)  Proficiency, word length, neighborhood effect

(B)  Frequency, age of acquisition (AoA), familiarity

(C)  McGurk effect, place and manner of articulation (PoA and MoA)

(D)  Language family, areal features, phoneme inventory size

Answer: (A, B)

60. Identify the language(s) that belong(s) to the Balkan Sprachbund.

(A)  Romanian

(B)  Bulgarian

(C)  Norwegian

(D)  Swedish

Answer: (A, B)

61. Consider the following tree structure, where A and C are co-referential. Similarly, E and H are co-referential. Identify the incorrect statement(s).

(A)  A binds C

(B)  C binds E

(C)  E binds D

(D)  A binds H

Answer: (B, C, D)

62. Which of the following statements is/are true for the pair – bird : cuckoo

(A)  Cuckoo is the hyponym of bird

(B)  Bird is the hypernym of cuckoo

(C)  Cuckoo is the hypernym of bird

(D)  Bird is the hyponym of cuckoo

Answer: (A, B)

63. In the sentences 1 through 4, in which one(s) a) entails b)?

(A)  1 a) entails 1 b)

(B)  2 a) entails 2 b)

(C)  3 a) entails 3 b)

(D)  4 a) entails 4 b)

Answer: (C, D)

64. The following figure depicts a spectrum of a vowel (dashed line), where F0 = 150 Hz and F0 = H1. The harmonics are indicated from H1 to H11. From the figure, the frequency (in Hz) of the second formant (F2) of this vowel is _______.

Answer: (1350 to 1350)

65. The number of core arguments associated with the ditransitive verb ‘give’ is _______.

Answer: (3 to 3)

XH : Humanities & Social Sciences

General Aptitude

Q.1 – Q.5 Carry ONE mark each.

1. Rafi told Mary, “I am thinking of watching a film this weekend.”

The following reports the above statement in indirect speech:

Rafi told Mary that he _______ of watching a film that weekend.

(A)  thought

(B)  is thinking

(C)  am thinking

(D)  was thinking

Answer: (D)

2. Permit : _______ : : Enforce : Relax

(By word meaning)

(A)  Allow

(B)  Forbid

(C)  License

(D)  Reinforce

Answer: (B)

3. Given a fair six-faced dice where the faces are labelled ‘1’, ‘2’, ‘3’, ‘4’, ‘5’, and ‘6’, what is the probability of getting a ‘1’ on the first roll of the dice and a ‘4’ on the second roll?

(A)  1/36

(B)  1/6

(C)  5/6

(D)  1/3

Answer: (A)

4. A recent survey shows that 65% of tobacco users were advised to stop consuming tobacco. The survey also shows that 3 out of 10 tobacco users attempted to stop using tobacco.

Based only on the information in the above passage, which one of the following options can be logically inferred with certainty?

(A)  A majority of tobacco users who were advised to stop consuming tobacco made an attempt to do so.

(B)  A majority of tobacco users who were advised to stop consuming tobacco did not attempt to do so.

(C)  Approximately 30% of tobacco users successfully stopped consuming tobacco.

(D)  Approximately 65% of tobacco users successfully stopped consuming tobacco.

Answer: (B)

5. How many triangles are present in the given figure?

(A)  12

(B)  16

(C)  20

(D)  24

Answer: (C)

Q.6 – Q.10 Carry TWO marks Each

6. Students of all the departments of a college who have successfully completed the registration process are eligible to vote in the upcoming college elections. However, by the time the due date for registration was over, it was found that suprisingly none of the students from the Department of Human Sciences had completed the registration process.

Based only on the information provided above, which one of the following sets of statement(s) can be logically inferred with certainty?

(i) All those students who would not be eligible to vote in the college elections would certainly belong to the Department of Human Sciences.

(ii) None of the students from departments other than Human Sciences failed to complete the registration process within the due time.

(iii) All the eligible voters would certainly be students who are not from the Department of Human Sciences.

(A)  (i) and (ii)

(B)  (i) and (iii)

(C)  only (i)

(D)  only (iii)

Answer: (D)

7. Which one of the following options represents the given graph?

(A)  f(x) = x22|x|

(B)  f(x) = x 2|x|

(C)  f(x) = |x|2x

(D)  f(x) = x2x

Answer: (B)

8. Which one of the options does NOT describe the passage below or follow from it?

We tend to think of cancer as a ‘modern’ illness because its metaphors are so modern. It is a disease of overproduction, of sudden growth, a growth that is unstoppable, tipped into the abyss of no control. Modern cell biology encourages us to imagine the cell as a molecular machine. Cancer is that machine unable to quench its intial command (to grow) and thus transform into an indestructible, self-propelled automaton.

[Adapted from The Emperor of All Maladies by Siddhartha Mukherjee]

(A)  It is a reflection of why cancer seems so modern to most of us.

(B)  It tells us that modern cell biology uses and promotes metaphors of machinery.

(C)  Modern cell biology encourages metaphors of machinery, and cancer is often imagined as a machine.

(D)  Modern cell biology never uses figurative language, such as metaphors, to describe or explain anything.

Answer: (D)

9. The digit in the unit’s place of the product 3999 × 71000 is ________.

(A)  7

(B)  1

(C)  3

(D)  9

Answer: (A)

10. A square with sides of length 6 cm is given. The boundary of the shaded region is defined by two semi-circles whose diameters are the sides of the square, as shown.

The area of the shaded region is _______ cm2.

(A)  6π

(B)  18

(C)  20

(D)  9π

Answer: (B)

Reasoning and Comprehension (XH-B1)

XH-B1: Q.11 – Q.17 Carry ONE mark Each

11. Which word below best describes the idea of being both Spineless and Cowardly?

(A)  Pusillanimous

(B)  Unctuous

(C)  Obsequious

(D)  Reticent

Answer: (A)

12. Choose the right preposition to fill up the blank:

The whole family got together ___ Diwali

(A)  of

(B)  at

(C)  in

(D)  till

Answer: (B)

13. Select the correct option to fill in all the blanks to complete the passage:

The (i) _______ factor amid this turbulence has been the (ii) ________ of high-octane, action-oriented films such as RRR, K.G.F: Chapter 2 and Pushpa from film industries in the south of the country. Traditionally, films made in the south have done well in their own (iii) _________. But increasingly, their dubbed versions have performed well in the Hindi heartland, with collections (iv) ________ those of their Bollywood counterparts.

(A)  (i) disheartening (ii) failure (iii) channels (iv) matching

(B)  (i) redeeming (ii) outperformance (iii) geographies (iv) eclipsing

(C)  (i) shocking (ii) underperformance (iii) cinemas (iv) below

(D)  (i) humbling (ii) bombing (iii) theatres (iv) falling behind

Answer: (B)

14. The following passage consists of 6 sentences. The first and sixth sentences of the passage are at their correct positions, while the middle four sentences (represented by 2, 3, 4, and 5) are jumbled up.

Choose the correct sequence of the sentences so that they form a coherent paragraph:

(1) Most obviously, mobility is taken to be a geographical as well as a social phenomenon.

(2) Much of the social mobility literature regarded society as a uniform surface and failed to register the geographical intersections of region, city and place, with the social categories of class, gender and ethnicity.

(3) The existing sociology of migration is incidentally far too limited in its concerns to be very useful here.

(4) Further, I am concerned with the flows of people within, but especially beyond, the territory of each society, and how these flows may relate to many different desires, for work, housing, leisure, religion, family relationships, criminal gain, asylum seeking and so on.

(5) Moreover, not only people are mobile but so too are many ‘objects’.

(6) I show that sociology’s recent development of a ‘sociology of objects’ needs to be taken further and that the diverse flows of objects across societal borders and their intersections with the multiple flows of people are hugely significant.

(A)  3, 2, 5, 4

(B)  2, 3, 4, 5

(C)  5, 4, 3, 2

(D)  4, 2, 5, 3

Answer: (B)

15. The population of a country increased by 5% from 2020 to 2021. Then, the population decreased by 5% from 2021 to 2022. By what percentage did the population change from 2020 to 2022?

(A)  -0.25%

(B)  0%

(C)  2.5%

(D)  10.25%

Answer: (A)

16. The words Thin: Slim: Slender are related in some way. Identify the correct option(s) that reflect(s) the same relationship:

(A)  Fat: Plump: Voluptuous

(B)  Short: Small: Petite

(C)  Tall: Taller: Tallest

(D)  Fair: Dark: Wheatish

Answer: (A, B)

17. A pandemic like situation hit the country last year, resulting in loss of human life and economic depression. To improve the condition of its citizens, the government made a series of emergency medical interventions and increased spending to revive the economy. In both these efforts, district administration authorities were actively involved.

Which of the following action(s) are plausible?

(A)  In future, the government can make district administration authorities responsible for protecting health of citizens and reviving the economy.

(B)  The government may set up a task force to review the post pandemic situation and ascertain the effectiveness of the measures taken.

(C)  The government may set up a committee to formulate a pandemic management program to minimize losses to life and economy in future.

(D)  The government may take population control measures to minimize pandemic related losses in future.

Answer: (B, C)

XH-B1: Q.18 – Q.26 Carry TWO marks Each

18. Six students, Arif, Balwinder, Chintu, David, Emon and Fulmoni appeared in the GATE-XH exam in 2022. Balwinder scores less than Chintu in XH-B1, but more than Arif in XH-C1. David scores more than Balwinder in XH-C1, and more than Chintu in XH-B1. Emon scores less than David, but more than Fulmoni in XH-B1. Fulmoni scores more than David in XH-C1. Arif scores less than Emon, but more than Fulmoni in XH-B1. Who scores highest in XH-B1?

(A)  Fulmoni

(B)  Emon

(C)  David

(D)  Chintu

Answer: (C)

19. Select the correct relation between E and F.

(A)  E > F

(B)  E < F

(C)  E = F

(D)  E < -F

Answer: (B)

20. A code language is formulated thus:

Vowels in the original word are replaced by the next vowel from the list of vowels, A-E-I-O-U (For example, E is replaced by I and U is replaced by A). Consonants in the original word are replaced by previous consonant (For example, T is replaced by S and V is replaced by T).

Then how does the word, GOODMORNING appear in the coded language?

(A)  HUUFNUSPOPH

(B)  FIICLIQMEMF

(C)  FUUCLUQMOMF

(D)  HEEDATTACRH

Answer: (C)

21. The stranger is by nature no “owner of soil” — soil not only in the physical, but also in the figurative sense of a life-substance, which is fixed, if not in a point in space, at least in an ideal point of the social environment. Although in more intimate relations, he may develop all kinds of charm and significance, as long as he is considered a stranger in the eyes of the other, he is not an “owner of soil.” Restriction to intermediary trade, and often (as though sublimated from it) to pure finance, gives him the specific character of mobility. If mobility takes place within a closed group, it embodies that synthesis of nearness and distance which constitutes the formal position of the stranger. For, the fundamentally mobile person comes in contact, at one time or another, with every individual, but is not organically connected, through established ties of kinship, locality, and occupation, with any single one.

What assumptions can be made about the stranger from the passage above?

(A)  The stranger can become an owner of soil through developing all kinds of charm in more intimate relations.

(B)  The stranger cannot become an owner of soil either in the physical or psychological sense.

(C)  The stranger can become an owner of soil through establishing ties of kinship and so on.

(D)  The stranger might become an owner of soil in the physical sense but not in the psychological

Answer: (B)

22. L is the only son of A and S. S has one sibling, B, who is married to L’s aunt, K. B is the only son of D. How are L and D related?

Select the possible option(s):

(A)  Grandchild and Paternal Grandfather

(B)  Grandchild and Maternal Grandfather

(C)  Grandchild and Paternal Grandmother

(D)  Grandchild and Maternal Grandmother

Answer: (B, D)

23. Five segments of a sentence are given below. The first and fifth segments are at their correct positions, while the middle three segments (represented by 2, 3, and 4) are jumbled up. Choose the correct order of the segments so that they form a coherent sentence:

(1) Consumed multitudes are jostling and shoving inside me

(2) and guided only by the memory of a large white bedsheet with a roughly circular hole some seven inches in diameter cut into the center,

(3) clutching at the dream of that holey, mutilated square of linen, which is my talisman, my open-sesame,

(4) I must commence the business of remaking my life from the point at which it really began,

(5) some thirty-two years before anything as obvious, as present, as my clock-ridden, crime-stained birth.

(A)  2 – 3 – 4

(B)  3 – 2– 4

(C)  4 – 2– 3

(D)  4 – 3 – 2

Answer: (A)

24. “I told you the truth,” I say yet again, “Memory’s truth, because memory has its own special kind. It selects, eliminates, alters, exaggerates, minimizes, glorifies, and vilifies also; but in the end it creates its own reality, its heterogeneous but usually coherent versions of events; and no sane human being ever trusts someone else’s version more than his own.”

What are the different ways in which ‘truth’ can be understood from the passage?

(A)  Truth is what can be verified by hard empirical evidence.

(B)  Truth is based on what can be perceived by the senses.

(C)  Truth is the product of memory that is fallible, selective and slanted.

(D)  Truth is contingent on the observer and can only be partial.

Answer: (C, D)

25. A firm needs both skilled labour and unskilled labour for the production of cloth. The wage of skilled labour is Rs. 40,000 per month, and that of unskilled labour is Rs. 15,000 per month. The total wage bill of the firm for the production of cloth is Rs. 23,75,000 in a month for 100 labour. How many skilled labour are employed by the firm (in Integer)?

Answer: (35 to 35)

26. Select the odd word and write the option number as answer:

(1) Lek (2) Zloty (3) Diner (4) Drachma (5) Real

Answer: (3 to 3)

Philosophy – C4

XH-C4: Q.27 – Q.44 Carry ONE mark Each

27. In Sāṅkhya philosophy ‘mind’ (manas) is an evolute of ______.

(A)  Prakṛti

(B)  Puruṣa

(C)  Three guṇas

(D)  Puruṣa and Prakṛti

Answer: (A)

28. Which one of the following is the manifestation of the Absolute Spirit in Hegel’s Phenomenology of Spirit?

(A)  The devotion of a church congregation

(B)  The knowledge of a natural scientist

(C)  The self-transparency of a society of free individuals

(D)  The mystical insight of an enlightened sage

Answer: (C)

29. The Cārvāka system accepts the following puruṣārthas:

(A)  Artha and Kāma

(B)  Dharma, Kāma and Mokṣa

(C)  Mokṣa and Dharma

(D)  Artha, Kāma and Mokṣa

Answer: (A)

30. In Taittirīya Upaniṣad there is a discussion of five sheaths (pañca-koṣa) in which the individual self is encased. Which one of the following is the sheath (koṣa) of knowledge and intelligence?

(A)  Vijñānamaya-koṣa

(B)  Prāṇamaya-koṣa

(C)  Manomaya-koṣa

(D)  Ᾱnandamaya-koṣa

Answer: (A)

31. According to Swami Vivekananda, ‘Universal Religion’ would consist in recognising that there are different ways of approaching the religious object. The watch-word for universal religion is ___________.

(A)  Tolerance

(B)  Acceptance

(C)  Submission

(D)  Adoration

Answer: (B)

32. In his ‘The Concept of the Absolute and Its Alternative Forms’, K. C. Bhattacharyya says, “… consciousness is of three kinds – knowing, feeling and willing…” Which one of the following is the ‘absolute’ of ‘knowing’?

(A)  Truth

(B)  Beauty

(C)  Goodness

(D)  Bliss

Answer: (A)

33. Which one of the following is NOT considered a pramāṇa by the Prābhākara Mīmāṃsaka?

(A)  Anupalabdhi (Non-apprehension)

(B)  Anumāna (Inference)

(C)  Arthāpatti (Postulation/ Presumption)

(D)  Upamāna (Comparison/ Analogy)

Answer: (A)

34. Marx introduces the concept of ‘commodity fetishism’ in Capital. Which one of the following is a correct description of the concept?

(A)  The reflection of human spiritual capacities in the material product

(B)  The reflection of human needs in the material product

(C)  The reflection of the social character of human labour in the material product

(D)  The reflection of the dignity of human labour in the material product

Answer: (C)

35. W.V. O. Quine famously writes in ‘Two Dogmas of Empiricism’:

(A)  No statement is immune to revision

(B)  Only analytic statements are immune to revision

(C)  Only synthetic statements are immune to revision

(D)  Only the statements of empirical sciences are immune to revision

Answer: (A)

36. Comparing the thoughts of Heraclitus and Parmenides, we can say that:

(A)  Both assert that only the soul (psuche) truly exists

(B)  Heraclitus asserts the unity underlying the plurality of things, whereas Parmenides denies the plurality of things altogether

(C)  Heraclitus asserts the infinity of the cosmos, whereas Parmenides asserts its finitude

(D)  Heraclitus asserts the incomprehensibility of the plurality of nature, while Parmenides asserts the fundamental comprehensibility of the plurality of nature

Answer: (B)

37. According to Heidegger’s Being and Time, the ontological difference is:

(A)  The distinction between Being (Sein) and beings (Seiende)

(B)  The distinction between Being (Sein) and Being-there (Dasein)

(C)  The distinction between beings (Seiende) and Being-there (Dasein)

(D)  The difference between Being (Sein) and Non-Being (Nichtsein)

Answer: (A)

38. In Plato’s Republic, the virtue of moderation is present:

(A)  Only in the guardians

(B)  In the auxiliaries and guardians

(C)  Only in the money makers

(D)  Throughout the republic

Answer: (D)

39. In Kāśmīra Śaivism, Śiva is the only reality, the one without a second. Which among the following is/are the other name[s] for Kāśmīra Śaivism?

(A)  Pratyabhijñā

(B)  Trika

(C)  Spanda

(D)  Vīra-śaivism

Answer: (A, B, C)

40. In ‘Democracy’, B. R. Ambedkar lays out certain fundamental assumptions about his conception of democracy. They include:

(A)  Adult suffrage and frequent elections are no bar against the governing class reaching places of power and authority

(B)  Servile classes volunteering to elect members of the governing class as their rulers is a sign of a thriving democracy

(C)  Servile classes in some countries may need other safeguards beside adult suffrage to oust the governing class from the seat of authority

(D)  Existence of the governing class is consistent with democracy and self-government

Answer: (A, C)

41. Read the following passage carefully and answer the question:

My uniform experience has convinced me that there is no other God than Truth…That is why my devotion to Truth has drawn me into the field of politics; and I can say without the slightest hesitation, yet in all humility, that those who say that religion has nothing to do with politics do not know what religion means. Identification with everything that lives is impossible without self-purification; without self-purification the observance of the law of Ahimsa must remain an empty dream; God can never be realized by one who is not pure of heart.

– M. K. Gandhi, An Autobiography or the Story of my Experiments with Truth, p. 615

Which among the following are NOT in conformity with the above passage?

(A)  God is Truth

(B)  Devotion to God, which is Truth, prompted Gandhi to enter politics

(C)  Religion and politics should be separated

(D)  Ahimsa and politics do not go hand in hand

Answer: (C, D)

42. Soli likes either logic or biology. If Soli likes logic, then he is not a happy person. Neither Soli nor Rupinder likes biology.

Which among the following can be concluded from the premises given here?

(A)  Soli is not a happy person.

(B)  Rupinder is not a happy person.

(C)  Rupinder is a happy person.

(D)  Soli likes logic.

Answer: (A, D)

43. On the basis of Aristotle’s Nicomachean Ethics, we can say the following about virtue:

(A)  We can know the virtue of something only if we understand the function (ergon) of that thing

(B)  Virtue entails deliberation

(C)  We become virtuous by simply knowing what virtue is

(D)  Virtue has the nature of a mean between two extremes in most cases

Answer: (A, B, D)

44. According to Hume’s An Inquiry Concerning Human Understanding, the following is/are the principle[s] governing the connection of ideas:

(A)  Resemblance

(B)  Contiguity in space and time

(C)  Juxtaposition

(D)  Cause and Effect

Answer: (A, B, D)

XH-C4: Q.45– Q.65 Carry TWO marks Each

45. In his Yogasūtra, Patañjali mentions five kinds of afflictions (kleśas). Which one of the following is NOT among the five afflictions?

(A)  Mātsarya (competition/ rivalry)

(B)  Avidyā (false knowledge)

(C)  Rāga (attachment)

(D)  Asmitā (egoism)

Answer: (A)

46. According to Jaina philosophy, all substances (dravya) but one have extension in space (astikāya). That one substance which has no extension in space (anastikāya) is ___.

(A)  Time (kāla)

(B)  Space (ākāśa)

(C)  Rest (adharma)

(D)  Motion (dharma)

Answer: (A)

47. Husserl’s fifth Cartesian Meditation is founded upon the realization that the reduction to my transcendental sphere of ownness:

(A)  Does not fundamentally cut me off from the other

(B)  Fatefully cuts me off from the other

(C)  Can never be completely achieved

(D)  Is not necessary to understand the constitution of the objective world

Answer: (A)

48. According to the Nyāya system, the argument “A sparrow is a bird, since it has wings” would have an inferential defect (hetvābhāsa) called _______.

(A)  Svarūpāsiddhi (unestablished in respect of itself)

(B)  Āśrayāsiddhi (unestablished in respect of abode)

(C)  Sādhāraṇa-anaikāntika (common strayer)

(D)  Asādhāraṇa-anaikāntika (uncommon strayer)

Answer: (C)

49. Consider the following sentence: ‘Dhavala is a white cow.’ For the Vaiśeṣika, the meaning (artha) of the sentence consists of the following padārthas:

(A)  Action (karma), Substance (dravya) and Unique particular (viśeṣa) only

(B)  Substance (dravya), Universal (sāmānya) and Quality (guṇa) only

(C)  Substance (dravya), Universal (sāmānya), Inherence (samavāya) and Quality (guṇa) only

(D)  Action (karma), Substance (dravya), Unique particular (viśeṣa) and Absence (abhāva) only

Answer: (C)

50. According to the theory advocated by G. Frege in his ‘On Sense and Reference’, the expression ‘the largest prime number’ would have ___________.

(A)  Both a sense and a reference

(B)  Only sense, and no reference

(C)  Only reference, and no sense

(D)  Neither sense, nor reference

Answer: (B)

51. In the Phaedo, Plato’s Socrates develops a novel way of understanding the beauty of things. He tells us:

(A)  The beauty of things is caused by nothing other than a combination of the colour, the shape and the size of a thing

(B)  The beauty of things is simply the effect of that thing on the eyes of the observer

(C)  If things are beautiful then it is due to the presence (parousia) of the beautiful in itself

(D)  The beauty of things is simply an illusion; only the beautiful exists in itself by itself (auto kath’ auto)

Answer: (C)

52. Descartes postulates the evil genius in his Meditations to deny the certainty of which statements?

(I) There is a table lamp to the left of the desk at which I am sitting.

(II) The sum of the angles of a triangle is 180 degrees.

(III) Sodium has only one electron in its outermost shell.

(IV) Plants absorb nourishment from the soil by means of their roots.

(V) I wish you were here.

(VI) I can see my friend in the 10th floor window of that building from where I stand on the road here below.

(VII) I am thinking of my dear mother.

(VIII) 2+2=4.

(A)  IV, III and V

(B)  I, II and III

(C)  V and VI

(D)  II and VIII

Answer: (D)

53. Which of the following statement[s] is/are NOT true in relation to Kant’s concept of the will?

(A)  Every will, even the divine will is subject to imperatives

(B)  Only the human will is subject to imperatives

(C)  The human will is subject to the hypothetical imperative, whereas the divine will is subject to the categorical imperative

(D)  The human will is subject to the categorical imperative which comes from God

Answer: (A, C, D)

54. We see a bronze statue of Poseidon in the National Archaeological Museum of Athens. Which of the following would be [a] cause[s] of the statue for Aristotle if we read his Physics?

(A)  Bronze

(B)  The sculptor who produced it

(C)  The plan to have a bronze statue of Poseidon installed in a temple

(D)  The space in the temple in which the statue is installed

Answer: (A, B, C)

55. In the Buddhist theory of elements (dharmas), dharmas are the ultimate momentary elements of existence. The number of elements varies in different schools of Buddhism. Of the following alternatives, which pair[s] does/do NOT give us the respective number of dharmas accepted in Sautrāntika and Sarvāstivāda (Vaibhāṣika) schools?

(A)  75 and 43

(B)  43 and 75

(C)  43 and 0

(D)  0 and 43

Answer: (A, C, D)

56. According to the Advaita of Śaṅkara, the individual self (jīva) is:

(A)  Not ontologically different from Brahman

(B)  Empirically different from Brahman due to the limiting adjuncts of body, mind, senses etc.

(C)  Ontologically real and one with Brahman

(D)  A part of Brahman

Answer: (A, B, C)

57. Mohammad Iqbal’s views on the nature of ‘intuition’ would state:

(A)  Intuition is immediate knowledge of the Reality (God)

(B)  Intuitive experience is not only subjective but also objective

(C)  Intuition is a property of the heart

(D)  Intuition is the property of the mind and the intellect

Answer: (A, B, C)

58. Sandra Harding’s Standpoint Epistemology involves:

(A)  Adopting a feminist empiricist point of view that critiques the masculine underpinnings of western sciences

(B)  Recognizing that science is value free

(C)  Negating the value of objectivity in scientific research

(D)  Grounding distinctive feminist science in strongly objective accounts of the world

Answer: (D)

59. In J. S. Mill’s articulation of utilitarianism which of the following statements about justice are valid?

(A)  Standards of justice stand higher in the scale of social utility

(B)  The just and the expedient are divided by an imaginary distinction

(C)  Social duty can become so important as to overrule any one of general maxims of justice

(D)  Utilitarianism prioritises expediency over justice

Answer: (A, C)

60. According to Russell’s ‘On Denoting’, the proposition, ‘The prime number between 7 and 11 is NOT larger than 12’ would be true, if _______.

(A)  The proposition, ‘The prime number between 7 and 11 is larger than 12’ is false

(B)  The expression ‘the prime number between 7 and 11’ has a primary occurrence in the proposition

(C)  The expression ‘the prime number between 7 and 11’ has a secondary occurrence in the proposition

(D)  There were only one prime number between 7 and 11

Answer: (C, D)

61. (i) p ⊃ (q ∙ r)

(ii) ~(p ⊃ s)

Taking (i) and (ii) as premises, which of the following can be deduced?

(A)  q

(B)  r

(C)  s

(D)  ~p

Answer: (A, B)

62. ~q can be deduced from ~(p ⊃ (q ∨ r)) by using rules of propositional logic in the following sequence[s]:

(A)  Material Implication> De Morgan’s Theorem> Commutation> Simplification> De Morgan’s Theorem> Simplification

(B)  Conjunction> Addition> Modus Ponens> De Morgan’s Theorem

(C)  Transposition> Material Implication> Double Negation> De Morgan’s Theorem> Simplification> De Morgan’s Theorem> Simplification

(D)  Modus Tollens> Conjunction> Hypothetical Syllogism> Simplification

Answer: (A, C)

63. Read the passage below from Wittgenstein’s Philosophical Investigations carefully and answer the question.

I can think of no better expression to characterize these similarities than “family resemblances”; for the various resemblances between members of a family – build, features, colour of eyes, gait, temperament, and so on and so forth – overlap and crisscross in the same way. – And I shall say: ‘games’ form a family.

And likewise the kinds of number, for example, form a family. Why do we call something a “number”? Well, perhaps because it has a – direct – affinity with several things that have hitherto been called “number”; and this can be said to give it an indirect affinity with other things that we also call “numbers.” And we extend our concept of number, as in spinning a thread we twist fibre on fibre. And the strength of the thread resides not in the fact that some one fibre runs through its whole length but in the overlapping of many fibres.

But if someone wanted to say, “So there is something common to all these constructions – namely, the disjunction of all their common properties” – I’d reply: Now you are only playing with a word. One might as well say, “There is a Something that runs through the whole thread – namely, the continuous overlapping of these “fibres.”

– Ludwig Wittgenstein, Philosophical Investigations, Investigation No. 67

Which of the following statement[s] does Wittgenstein imply in the above passage?

(A)  The one fibre supposed to run through the length of the thread corresponds to what is supposed to be common to all numbers

(B)  The overlapping of the fibres corresponds to the family resemblance Wittgenstein is trying to explicate

(C)  It is absurd to insist that there is one thing common to all the members of the family

(D)  To describe a family resemblance is another way to describe the common property shared by all the family members

Answer: (A, B, C)

64. In Kant’s Critique of Pure Reason, the equation 7 + 5 = 12 is synthetic a priori and not simply analytic for the following reason[s].

(A)  A simple analysis of the concept of 5, the concept of 7 and the concept of addition does not give us the sum 12

(B)  With 7 as the starting point, we require an intuition such as 5 fingers or 5 marks on a page to reach upto 12

(C)  If 7 + 5 = 12 were an analytic proposition it would not need any intuition to know it

(D)  We arrive at the knowledge of 7 + 5 = 12 by imitating others adding 7 and 5

Answer: (A, B, C)

65. According to Sartre’s Being and Nothingness, which of following statement[s] is/ are true?

(A)  Bad faith is human beings’ denial of their own freedom

(B)  To assert the existence of nothingness is to deny the existence of human freedom

(C)  To assert the existence of nothingness is to assert the existence of human freedom

(D)  Being and nothingness self-evidently exclude each other

Answer: (A, C)

XH : Humanities & Social Sciences

General Aptitude

Q.1 – Q.5 Carry ONE mark each.

1. Rafi told Mary, “I am thinking of watching a film this weekend.”

The following reports the above statement in indirect speech:

Rafi told Mary that he _______ of watching a film that weekend.

(A)  thought

(B)  is thinking

(C)  am thinking

(D)  was thinking

Answer: (D)

2. Permit : _______ : : Enforce : Relax

(By word meaning)

(A)  Allow

(B)  Forbid

(C)  License

(D)  Reinforce

Answer: (B)

3. Given a fair six-faced dice where the faces are labelled ‘1’, ‘2’, ‘3’, ‘4’, ‘5’, and ‘6’, what is the probability of getting a ‘1’ on the first roll of the dice and a ‘4’ on the second roll?

(A)  1/36

(B)  1/6

(C)  5/6

(D)  1/3

Answer: (A)

4. A recent survey shows that 65% of tobacco users were advised to stop consuming tobacco. The survey also shows that 3 out of 10 tobacco users attempted to stop using tobacco.

Based only on the information in the above passage, which one of the following options can be logically inferred with certainty?

(A)  A majority of tobacco users who were advised to stop consuming tobacco made an attempt to do so.

(B)  A majority of tobacco users who were advised to stop consuming tobacco did not attempt to do so.

(C)  Approximately 30% of tobacco users successfully stopped consuming tobacco.

(D)  Approximately 65% of tobacco users successfully stopped consuming tobacco.

Answer: (B)

5. How many triangles are present in the given figure?

(A)  12

(B)  16

(C)  20

(D)  24

Answer: (C)

Q.6 – Q.10 Carry TWO marks Each

6. Students of all the departments of a college who have successfully completed the registration process are eligible to vote in the upcoming college elections. However, by the time the due date for registration was over, it was found that suprisingly none of the students from the Department of Human Sciences had completed the registration process.

Based only on the information provided above, which one of the following sets of statement(s) can be logically inferred with certainty?

(i) All those students who would not be eligible to vote in the college elections would certainly belong to the Department of Human Sciences.

(ii) None of the students from departments other than Human Sciences failed to complete the registration process within the due time.

(iii) All the eligible voters would certainly be students who are not from the Department of Human Sciences.

(A)  (i) and (ii)

(B)  (i) and (iii)

(C)  only (i)

(D)  only (iii)

Answer: (D)

7. Which one of the following options represents the given graph?

(A)  f(x) = x22|x|

(B)  f(x) = x 2|x|

(C)  f(x) = |x|2x

(D)  f(x) = x2x

Answer: (B)

8. Which one of the options does NOT describe the passage below or follow from it?

We tend to think of cancer as a ‘modern’ illness because its metaphors are so modern. It is a disease of overproduction, of sudden growth, a growth that is unstoppable, tipped into the abyss of no control. Modern cell biology encourages us to imagine the cell as a molecular machine. Cancer is that machine unable to quench its intial command (to grow) and thus transform into an indestructible, self-propelled automaton.

[Adapted from The Emperor of All Maladies by Siddhartha Mukherjee]

(A)  It is a reflection of why cancer seems so modern to most of us.

(B)  It tells us that modern cell biology uses and promotes metaphors of machinery.

(C)  Modern cell biology encourages metaphors of machinery, and cancer is often imagined as a machine.

(D)  Modern cell biology never uses figurative language, such as metaphors, to describe or explain anything.

Answer: (D)

9. The digit in the unit’s place of the product 3999 × 71000 is ________.

(A)  7

(B)  1

(C)  3

(D)  9

Answer: (A)

10. A square with sides of length 6 cm is given. The boundary of the shaded region is defined by two semi-circles whose diameters are the sides of the square, as shown.

The area of the shaded region is _______ cm2.

(A)  6π

(B)  18

(C)  20

(D)  9π

Answer: (B)

Reasoning and Comprehension (XH-B1)

XH-B1: Q.11 – Q.17 Carry ONE mark Each

11. Which word below best describes the idea of being both Spineless and Cowardly?

(A)  Pusillanimous

(B)  Unctuous

(C)  Obsequious

(D)  Reticent

Answer: (A)

12. Choose the right preposition to fill up the blank:

The whole family got together ___ Diwali

(A)  of

(B)  at

(C)  in

(D)  till

Answer: (B)

13. Select the correct option to fill in all the blanks to complete the passage:

The (i) _______ factor amid this turbulence has been the (ii) ________ of high-octane, action-oriented films such as RRR, K.G.F: Chapter 2 and Pushpa from film industries in the south of the country. Traditionally, films made in the south have done well in their own (iii) _________. But increasingly, their dubbed versions have performed well in the Hindi heartland, with collections (iv) ________ those of their Bollywood counterparts.

(A)  (i) disheartening (ii) failure (iii) channels (iv) matching

(B)  (i) redeeming (ii) outperformance (iii) geographies (iv) eclipsing

(C)  (i) shocking (ii) underperformance (iii) cinemas (iv) below

(D)  (i) humbling (ii) bombing (iii) theatres (iv) falling behind

Answer: (B)

14. The following passage consists of 6 sentences. The first and sixth sentences of the passage are at their correct positions, while the middle four sentences (represented by 2, 3, 4, and 5) are jumbled up.

Choose the correct sequence of the sentences so that they form a coherent paragraph:

(1) Most obviously, mobility is taken to be a geographical as well as a social phenomenon.

(2) Much of the social mobility literature regarded society as a uniform surface and failed to register the geographical intersections of region, city and place, with the social categories of class, gender and ethnicity.

(3) The existing sociology of migration is incidentally far too limited in its concerns to be very useful here.

(4) Further, I am concerned with the flows of people within, but especially beyond, the territory of each society, and how these flows may relate to many different desires, for work, housing, leisure, religion, family relationships, criminal gain, asylum seeking and so on.

(5) Moreover, not only people are mobile but so too are many ‘objects’.

(6) I show that sociology’s recent development of a ‘sociology of objects’ needs to be taken further and that the diverse flows of objects across societal borders and their intersections with the multiple flows of people are hugely significant.

(A)  3, 2, 5, 4

(B)  2, 3, 4, 5

(C)  5, 4, 3, 2

(D)  4, 2, 5, 3

Answer: (B)

15. The population of a country increased by 5% from 2020 to 2021. Then, the population decreased by 5% from 2021 to 2022. By what percentage did the population change from 2020 to 2022?

(A)  -0.25%

(B)  0%

(C)  2.5%

(D)  10.25%

Answer: (A)

16. The words Thin: Slim: Slender are related in some way. Identify the correct option(s) that reflect(s) the same relationship:

(A)  Fat: Plump: Voluptuous

(B)  Short: Small: Petite

(C)  Tall: Taller: Tallest

(D)  Fair: Dark: Wheatish

Answer: (A, B)

17. A pandemic like situation hit the country last year, resulting in loss of human life and economic depression. To improve the condition of its citizens, the government made a series of emergency medical interventions and increased spending to revive the economy. In both these efforts, district administration authorities were actively involved.

Which of the following action(s) are plausible?

(A)  In future, the government can make district administration authorities responsible for protecting health of citizens and reviving the economy.

(B)  The government may set up a task force to review the post pandemic situation and ascertain the effectiveness of the measures taken.

(C)  The government may set up a committee to formulate a pandemic management program to minimize losses to life and economy in future.

(D)  The government may take population control measures to minimize pandemic related losses in future.

Answer: (B, C)

XH-B1: Q.18 – Q.26 Carry TWO marks Each

18. Six students, Arif, Balwinder, Chintu, David, Emon and Fulmoni appeared in the GATE-XH exam in 2022. Balwinder scores less than Chintu in XH-B1, but more than Arif in XH-C1. David scores more than Balwinder in XH-C1, and more than Chintu in XH-B1. Emon scores less than David, but more than Fulmoni in XH-B1. Fulmoni scores more than David in XH-C1. Arif scores less than Emon, but more than Fulmoni in XH-B1. Who scores highest in XH-B1?

(A)  Fulmoni

(B)  Emon

(C)  David

(D)  Chintu

Answer: (C)

19. Select the correct relation between E and F.

(A)  E > F

(B)  E < F

(C)  E = F

(D)  E < -F

Answer: (B)

20. A code language is formulated thus:

Vowels in the original word are replaced by the next vowel from the list of vowels, A-E-I-O-U (For example, E is replaced by I and U is replaced by A). Consonants in the original word are replaced by previous consonant (For example, T is replaced by S and V is replaced by T).

Then how does the word, GOODMORNING appear in the coded language?

(A)  HUUFNUSPOPH

(B)  FIICLIQMEMF

(C)  FUUCLUQMOMF

(D)  HEEDATTACRH

Answer: (C)

21. The stranger is by nature no “owner of soil” — soil not only in the physical, but also in the figurative sense of a life-substance, which is fixed, if not in a point in space, at least in an ideal point of the social environment. Although in more intimate relations, he may develop all kinds of charm and significance, as long as he is considered a stranger in the eyes of the other, he is not an “owner of soil.” Restriction to intermediary trade, and often (as though sublimated from it) to pure finance, gives him the specific character of mobility. If mobility takes place within a closed group, it embodies that synthesis of nearness and distance which constitutes the formal position of the stranger. For, the fundamentally mobile person comes in contact, at one time or another, with every individual, but is not organically connected, through established ties of kinship, locality, and occupation, with any single one.

What assumptions can be made about the stranger from the passage above?

(A)  The stranger can become an owner of soil through developing all kinds of charm in more intimate relations.

(B)  The stranger cannot become an owner of soil either in the physical or psychological sense.

(C)  The stranger can become an owner of soil through establishing ties of kinship and so on.

(D)  The stranger might become an owner of soil in the physical sense but not in the psychological

Answer: (B)

22. L is the only son of A and S. S has one sibling, B, who is married to L’s aunt, K. B is the only son of D. How are L and D related?

Select the possible option(s):

(A)  Grandchild and Paternal Grandfather

(B)  Grandchild and Maternal Grandfather

(C)  Grandchild and Paternal Grandmother

(D)  Grandchild and Maternal Grandmother

Answer: (B, D)

23. Five segments of a sentence are given below. The first and fifth segments are at their correct positions, while the middle three segments (represented by 2, 3, and 4) are jumbled up. Choose the correct order of the segments so that they form a coherent sentence:

(1) Consumed multitudes are jostling and shoving inside me

(2) and guided only by the memory of a large white bedsheet with a roughly circular hole some seven inches in diameter cut into the center,

(3) clutching at the dream of that holey, mutilated square of linen, which is my talisman, my open-sesame,

(4) I must commence the business of remaking my life from the point at which it really began,

(5) some thirty-two years before anything as obvious, as present, as my clock-ridden, crime-stained birth.

(A)  2 – 3 – 4

(B)  3 – 2– 4

(C)  4 – 2– 3

(D)  4 – 3 – 2

Answer: (A)

24. “I told you the truth,” I say yet again, “Memory’s truth, because memory has its own special kind. It selects, eliminates, alters, exaggerates, minimizes, glorifies, and vilifies also; but in the end it creates its own reality, its heterogeneous but usually coherent versions of events; and no sane human being ever trusts someone else’s version more than his own.”

What are the different ways in which ‘truth’ can be understood from the passage?

(A)  Truth is what can be verified by hard empirical evidence.

(B)  Truth is based on what can be perceived by the senses.

(C)  Truth is the product of memory that is fallible, selective and slanted.

(D)  Truth is contingent on the observer and can only be partial.

Answer: (C, D)

25. A firm needs both skilled labour and unskilled labour for the production of cloth. The wage of skilled labour is Rs. 40,000 per month, and that of unskilled labour is Rs. 15,000 per month. The total wage bill of the firm for the production of cloth is Rs. 23,75,000 in a month for 100 labour. How many skilled labour are employed by the firm (in Integer)?

Answer: (35 to 35)

26. Select the odd word and write the option number as answer:

(1) Lek (2) Zloty (3) Diner (4) Drachma (5) Real

Answer: (3 to 3)

Psychology – C5

XH-C5: Q.27 – Q.44 Carry ONE mark Each

27. Blind spot in the retina contains _________.

(A)  only rod cells

(B)  only cone cells

(C)  both rod and cone cells

(D)  neither rod nor cone cells

Answer: (D)

28. Taking painkillers eliminates pain, increasing the likelihood that the person will take painkillers again. This is an example of __________.

(A)  negative punishment

(B)  positive reinforcement

(C)  negative reinforcement

(D)  positive punishment

Answer: (C)

29. When learning something new impairs the ability to retrieve information learnt earlier, it is known as ____________.

(A)  retroactive interference

(B)  proactive interference

(C)  tip-of-the-tongue phenomenon

(D)  recency effect

Answer: (A)

30. Iconic memory is a type of _________.

(A)  short term memory

(B)  sensory memory

(C)  semantic memory

(D)  working memory

Answer: (B)

31. Which one of the following components of language has to do with the practical or social aspects of communication with others?

(A)  Phonemes

(B)  Morphemes

(C)  Pragmatics

(D)  Syntax

Answer: (C)

32. The Yerkes-Dodson law states that _____________.

(A)  performance is affected by the level of arousal

(B)  frustration leads to aggression

(C)  self-concept helps us organize and remember information

(D)  changes in behaviour are a result of experiences that have happened frequently or recently

Answer: (A)

33. Transduction of mechanical energy into nerve impulses in the auditory system takes place in reponse to bending of the ____________.

(A)  pinna

(B)  hair cells

(C)  incus

(D)  malleus

Answer: (B)

34. Which one of the following theories states that emotion occurs as a result of physical arousal and labeling of the arousal based on cues from the surrounding environment?

(A)  Ekman-Friesen theory

(B)  Schachter-Singer theory

(C)  James-Lange theory

(D)  Cannon-Bard theory

Answer: (B)

35. Which one of the following is TRUE for the Pain-Gate control theory?

(A)  Gate is a physical structure

(B)  Activity of gate cannot be closed by non-pain signals

(C)  Substance P released into spinal cord does not activate other neurons that send messages through spinal gates

(D)  Pain signals must pass through a gate located in the spinal cord

Answer: (D)

36. Being treated with warmth and consideration by others only when one behaves as expected, is called as ___________.

(A)  openness to experience

(B)  conditional positive regard

(C)  unconditional positive regard

(D)  zone of proximal development

Answer: (B)

37. Which of the following is/are NOT the factor(s) of the Big Five Personality model?

(A)  Conscientiousness

(B)  Optimism

(C)  Humility

(D)  Extraversion

Answer: (B, C)

38. Which of the following is/are TRUE for creative individuals?

(A)  They are not very good at mental imagery

(B)  They are not afraid to be different

(C)  They do not value their independence

(D)  They are unconventional in their work

Answer: (B, D)

39. Which of the following is/are included under behavioural genetics studies in humans?

(A)  Selective breeding studies

(B)  Family studies

(C)  Twin studies

(D)  Adoption studies

Answer: (B, C, D)

40. Which of the following statements regarding locus of control is/are CORRECT?

(A)  It can be internal and/or external

(B)  It is associated with self-esteem

(C)  Internal locus of control is positively correlated with success

(D)  Regret has no association with locus of control

Answer: (A, B, C)

41. Which of the following is/are TRUE for nonparametric statistics?

(A)  It is often called distribution free statitstics

(B)  It is used to analyze interval data

(C)  It is used to analyze ordinal data

(D)  It compares groups in terms of means

Answer: (A, C)

42. According to the review of research on stress and immune system, which of the following statement(s) has/have substantial evidence?

(A)  There is a negative relationship between stress and functional immune measures

(B)  There is no measurable impact of stress on functional immune measures

(C)  Loneliness impacts the relationship between stress and functional immune measures

(D)  There is a positive relationship between long term stressors and functional immune measures

Answer: (A, C)

43. Which of the following influence(s) gender dysphoria?

(A)  Prenatal factors

(B)  Early childhood experiences

(C)  Socialization

(D)  Attention Deficit Hyperactivity Disorder

Answer: (A, B, C)

44. If the variance of a set of scores is 100, the standard deviation is ________ (in integer).

Answer: (10 to 10)

XH-C5: Q.45– Q.65 Carry TWO marks Each

45. Match the structures of the brain in the first column with their respective functions in the second column of the table given below.

(A)  P-(iii), Q-(i), R-(iv), S-(ii)

(B)  P-(i), Q-(iv), R-(ii), S-(iii)

(C)  P-(iv), Q-(iii), R-(ii), S-(i)

(D)  P-(iii), Q-(iv), R-(i), S-(ii)

Answer: (D)

46. Which one of the following is established by the replication of research studies?

(A)  Validity of the results

(B)  Reliability of the results

(C)  Interaction effect

(D)  Mediation effect

Answer: (B)

47. Match the theory/law in the first column with the corresponding explanation in the second column of the table given below.

(A)  P-(ii), Q-(iv), R-(i), S-(iii)

(B)  P-(iv), Q-(iii), R-(ii), S-(i)

(C)  P-(i), Q-(ii), R-(iv), S-(iii)

(D)  P-(iii), Q-(ii), R-(iv), S-(i)

Answer: (C)

48. Match the sleep disorders in the first column with the symptoms in the second column of the table given below.

(A)  P-(ii), Q-(iv), R-(i), S-(iii)

(B)  P-(ii), Q-(i), R-(iv), S-(iii)

(C)  P-(ii), Q-(i), R-(iii), S-(iv)

(D)  P-(iii), Q-(iv), R-(i), S-(ii)

Answer: (B)

49. __________ explains aging as a process of mutual withdrawal of individual and society, whereas _________ assumes positive correlation between activity and successful aging.

(A)  Decay theory, Engagement theory

(B)  Balance theory, Engagement theory

(C)  Disengagement theory, Activity theory

(D)  Withdrawal theory, Activity theory

Answer: (C)

50. Match the biases/effects in the first column with the descriptions in the second column of the table given below.

(A)  P-(i), Q-(iii), R-(ii), S-(iv)

(B)  P-(iv), Q-(iii), R-(i), S-(ii)

(C)  P-(iv), Q-(iii), R-(ii), S-(i)

(D)  P-(iv), Q-(i), R-(ii), S-(iii)

Answer: (B)

51. Match the theories in the first column with the central themes in the second column of the table given below.

(A)  P-(iv), Q-(i), R-(ii), S-(iii)

(B)  P-(iv), Q-(i), R-(iii), S-(ii)

(C)  P-(i), Q-(iv), R-(ii), S-(iii)

(D)  P-(ii), Q-(iii), R-(i), S-(iv)

Answer: (A)

52. Which one of the following would resolve the basic dilemma of the social psychologist?

(A)  Conducting carefully designed experiments with high internal validity

(B)  By replicating experiments and conducting some new experiments that have internal validity and others that have external validity

(C)  Conducting research exclusively in the field

(D)  Conducting applied rather than basic research

Answer: (B)

53. Match the research methods in the first column with their purposes in the second column of the table given below.

(A)  P-(i), Q-(iii), R-(ii), S-(iv)

(B)  P-(ii), Q-(iii), R-(iv), S-(i)

(C)  P-(i), Q-(iv), R-(ii), S-(iii)

(D)  P-(ii), Q-(iv), R-(i), S-(iii)

Answer: (D)

54. Which one of the following statements is TRUE according to Brehm’s reactance theory?

(A)  A mild sign prohibitting spitting would be more effective in preventing spitting than a strong sign

(B)  A strong sign would be more effective in preventing spitting than a mild one

(C)  Absence of any sign would be most effective in preventing spitting

(D)  Signs are irrelevant to spitting behaviour

Answer: (A)

55. According to the overjustifcation effect, which one of the following consequences would be TRUE for students who freely choose to study psychology, if marks are given for attendance?

(A)  Increased interest of students in the subject

(B)  Decreased interest of students in the subject

(C)  No effect on the interest of students in the subject

(D)  Increased interest in other social science subjects

Answer: (B)

56. Large rewards and severe punishments are examples of __________ justification for behaviour and result in _________ changes in attitude.

(A)  internal; big

(B)  external; big

(C)  internal; small

(D)  external; small

Answer: (D)

57. Which of the following therapies is/are based on classical conditioning?

(A)  Systematic desensitization

(B)  Aversion therapy

(C)  Cognitive behaviour therapy

(D)  Rational emotive behaviour therapy

Answer: (A, B)

58. Which of the following characterize(s) Wernicke’s aphasia?

(A)  Inability to comprehend spoken words

(B)  Inability to understand the meaning of words

(C)  Inability to speak grammatically correct language

(D)  Inability to write and understand the symbols that represent speech sounds

Answer: (A, B, C, D)

59. Which of the following is/are the component(s) of Theory of Mind?

(A)  Understanding that people can have false beliefs

(B)  Distinguishing faces and smells

(C)  Recognizing that others have mental states

(D)  Having a vast vocabulary to express one’s thoughts

Answer: (A, C)

60. Which of the following would NOT be effective in minimizing groupthink?

(A)  The leader being impartial to all ideas no matter what they are

(B)  The leader reminding the team that everyone will be held responsible for the decision of the group

(C)  The leader taking all decisions on behalf of the group

(D)  The leader arriving at a consensus quickly

Answer: (C, D)

61. Which of the following describe(s) the standard error of the mean?

(A)  It is the standard deviation of the sampling distribution of the mean

(B)  It reflects the accuracy with which sample means estimate the population mean

(C)  It is the difference between mean and standard deviation of a distribution

(D)  It is the standard deviation of a stratified sample

Answer: (A, B)

62. Which of the following describe(s) organizational commitment?

(A)  A strong desire to remain a member of the organization

(B)  Willingness to exert high level of effort on behalf of the organization

(C)  A definite belief in and acceptance of values and goals of the organization

(D)  High turnover intention of the employees of the organization

Answer: (A, B, C)

63. Prejudice is supported by the human tendency to categorize into in-groups and out-groups. Prejudice is supported by which of the following processes?

(A)  The way we think about others

(B)  The way we assign meaning to others behaviour

(C)  By following intellectual pursuits

(D)  By working towards a common goal

Answer: (A, B)

64. Which of the following is/are feature(s) of clinical phobia?

(A)  The fear must be persistent

(B)  The fear must be a source of significant distress

(C)  The fear is rational

(D)  The fear is usually perceived as unwarranted

Answer: (A, B, D)

65. The value of F calculated from the data given in the table below is ________ (rounded off to one decimal place).

Answer: (3.6 to 3.6)

XH : Humanities & Social Sciences

General Aptitude

Q.1 – Q.5 Carry ONE mark each.

1. Rafi told Mary, “I am thinking of watching a film this weekend.”

The following reports the above statement in indirect speech:

Rafi told Mary that he _______ of watching a film that weekend.

(A)  thought

(B)  is thinking

(C)  am thinking

(D)  was thinking

Answer: (D)

2. Permit : _______ : : Enforce : Relax

(By word meaning)

(A)  Allow

(B)  Forbid

(C)  License

(D)  Reinforce

Answer: (B)

3. Given a fair six-faced dice where the faces are labelled ‘1’, ‘2’, ‘3’, ‘4’, ‘5’, and ‘6’, what is the probability of getting a ‘1’ on the first roll of the dice and a ‘4’ on the second roll?

(A)  1/36

(B)  1/6

(C)  5/6

(D)  1/3

Answer: (A)

4. A recent survey shows that 65% of tobacco users were advised to stop consuming tobacco. The survey also shows that 3 out of 10 tobacco users attempted to stop using tobacco.

Based only on the information in the above passage, which one of the following options can be logically inferred with certainty?

(A)  A majority of tobacco users who were advised to stop consuming tobacco made an attempt to do so.

(B)  A majority of tobacco users who were advised to stop consuming tobacco did not attempt to do so.

(C)  Approximately 30% of tobacco users successfully stopped consuming tobacco.

(D)  Approximately 65% of tobacco users successfully stopped consuming tobacco.

Answer: (B)

5. How many triangles are present in the given figure?

(A)  12

(B)  16

(C)  20

(D)  24

Answer: (C)

Q.6 – Q.10 Carry TWO marks Each

6. Students of all the departments of a college who have successfully completed the registration process are eligible to vote in the upcoming college elections. However, by the time the due date for registration was over, it was found that suprisingly none of the students from the Department of Human Sciences had completed the registration process.

Based only on the information provided above, which one of the following sets of statement(s) can be logically inferred with certainty?

(i) All those students who would not be eligible to vote in the college elections would certainly belong to the Department of Human Sciences.

(ii) None of the students from departments other than Human Sciences failed to complete the registration process within the due time.

(iii) All the eligible voters would certainly be students who are not from the Department of Human Sciences.

(A)  (i) and (ii)

(B)  (i) and (iii)

(C)  only (i)

(D)  only (iii)

Answer: (D)

7. Which one of the following options represents the given graph?

(A)  f(x) = x22|x|

(B)  f(x) = x 2|x|

(C)  f(x) = |x|2x

(D)  f(x) = x2x

Answer: (B)

8. Which one of the options does NOT describe the passage below or follow from it?

We tend to think of cancer as a ‘modern’ illness because its metaphors are so modern. It is a disease of overproduction, of sudden growth, a growth that is unstoppable, tipped into the abyss of no control. Modern cell biology encourages us to imagine the cell as a molecular machine. Cancer is that machine unable to quench its intial command (to grow) and thus transform into an indestructible, self-propelled automaton.

[Adapted from The Emperor of All Maladies by Siddhartha Mukherjee]

(A)  It is a reflection of why cancer seems so modern to most of us.

(B)  It tells us that modern cell biology uses and promotes metaphors of machinery.

(C)  Modern cell biology encourages metaphors of machinery, and cancer is often imagined as a machine.

(D)  Modern cell biology never uses figurative language, such as metaphors, to describe or explain anything.

Answer: (D)

9. The digit in the unit’s place of the product 3999 × 71000 is ________.

(A)  7

(B)  1

(C)  3

(D)  9

Answer: (A)

10. A square with sides of length 6 cm is given. The boundary of the shaded region is defined by two semi-circles whose diameters are the sides of the square, as shown.

The area of the shaded region is _______ cm2.

(A)  6π

(B)  18

(C)  20

(D)  9π

Answer: (B)

Reasoning and Comprehension (XH-B1)

XH-B1: Q.11 – Q.17 Carry ONE mark Each

11. Which word below best describes the idea of being both Spineless and Cowardly?

(A)  Pusillanimous

(B)  Unctuous

(C)  Obsequious

(D)  Reticent

Answer: (A)

12. Choose the right preposition to fill up the blank:

The whole family got together ___ Diwali

(A)  of

(B)  at

(C)  in

(D)  till

Answer: (B)

13. Select the correct option to fill in all the blanks to complete the passage:

The (i) _______ factor amid this turbulence has been the (ii) ________ of high-octane, action-oriented films such as RRR, K.G.F: Chapter 2 and Pushpa from film industries in the south of the country. Traditionally, films made in the south have done well in their own (iii) _________. But increasingly, their dubbed versions have performed well in the Hindi heartland, with collections (iv) ________ those of their Bollywood counterparts.

(A)  (i) disheartening (ii) failure (iii) channels (iv) matching

(B)  (i) redeeming (ii) outperformance (iii) geographies (iv) eclipsing

(C)  (i) shocking (ii) underperformance (iii) cinemas (iv) below

(D)  (i) humbling (ii) bombing (iii) theatres (iv) falling behind

Answer: (B)

14. The following passage consists of 6 sentences. The first and sixth sentences of the passage are at their correct positions, while the middle four sentences (represented by 2, 3, 4, and 5) are jumbled up.

Choose the correct sequence of the sentences so that they form a coherent paragraph:

(1) Most obviously, mobility is taken to be a geographical as well as a social phenomenon.

(2) Much of the social mobility literature regarded society as a uniform surface and failed to register the geographical intersections of region, city and place, with the social categories of class, gender and ethnicity.

(3) The existing sociology of migration is incidentally far too limited in its concerns to be very useful here.

(4) Further, I am concerned with the flows of people within, but especially beyond, the territory of each society, and how these flows may relate to many different desires, for work, housing, leisure, religion, family relationships, criminal gain, asylum seeking and so on.

(5) Moreover, not only people are mobile but so too are many ‘objects’.

(6) I show that sociology’s recent development of a ‘sociology of objects’ needs to be taken further and that the diverse flows of objects across societal borders and their intersections with the multiple flows of people are hugely significant.

(A)  3, 2, 5, 4

(B)  2, 3, 4, 5

(C)  5, 4, 3, 2

(D)  4, 2, 5, 3

Answer: (B)

15. The population of a country increased by 5% from 2020 to 2021. Then, the population decreased by 5% from 2021 to 2022. By what percentage did the population change from 2020 to 2022?

(A)  -0.25%

(B)  0%

(C)  2.5%

(D)  10.25%

Answer: (A)

16. The words Thin: Slim: Slender are related in some way. Identify the correct option(s) that reflect(s) the same relationship:

(A)  Fat: Plump: Voluptuous

(B)  Short: Small: Petite

(C)  Tall: Taller: Tallest

(D)  Fair: Dark: Wheatish

Answer: (A, B)

17. A pandemic like situation hit the country last year, resulting in loss of human life and economic depression. To improve the condition of its citizens, the government made a series of emergency medical interventions and increased spending to revive the economy. In both these efforts, district administration authorities were actively involved.

Which of the following action(s) are plausible?

(A)  In future, the government can make district administration authorities responsible for protecting health of citizens and reviving the economy.

(B)  The government may set up a task force to review the post pandemic situation and ascertain the effectiveness of the measures taken.

(C)  The government may set up a committee to formulate a pandemic management program to minimize losses to life and economy in future.

(D)  The government may take population control measures to minimize pandemic related losses in future.

Answer: (B, C)

XH-B1: Q.18 – Q.26 Carry TWO marks Each

18. Six students, Arif, Balwinder, Chintu, David, Emon and Fulmoni appeared in the GATE-XH exam in 2022. Balwinder scores less than Chintu in XH-B1, but more than Arif in XH-C1. David scores more than Balwinder in XH-C1, and more than Chintu in XH-B1. Emon scores less than David, but more than Fulmoni in XH-B1. Fulmoni scores more than David in XH-C1. Arif scores less than Emon, but more than Fulmoni in XH-B1. Who scores highest in XH-B1?

(A)  Fulmoni

(B)  Emon

(C)  David

(D)  Chintu

Answer: (C)

19. Select the correct relation between E and F.

(A)  E > F

(B)  E < F

(C)  E = F

(D)  E < -F

Answer: (B)

20. A code language is formulated thus:

Vowels in the original word are replaced by the next vowel from the list of vowels, A-E-I-O-U (For example, E is replaced by I and U is replaced by A). Consonants in the original word are replaced by previous consonant (For example, T is replaced by S and V is replaced by T).

Then how does the word, GOODMORNING appear in the coded language?

(A)  HUUFNUSPOPH

(B)  FIICLIQMEMF

(C)  FUUCLUQMOMF

(D)  HEEDATTACRH

Answer: (C)

21. The stranger is by nature no “owner of soil” — soil not only in the physical, but also in the figurative sense of a life-substance, which is fixed, if not in a point in space, at least in an ideal point of the social environment. Although in more intimate relations, he may develop all kinds of charm and significance, as long as he is considered a stranger in the eyes of the other, he is not an “owner of soil.” Restriction to intermediary trade, and often (as though sublimated from it) to pure finance, gives him the specific character of mobility. If mobility takes place within a closed group, it embodies that synthesis of nearness and distance which constitutes the formal position of the stranger. For, the fundamentally mobile person comes in contact, at one time or another, with every individual, but is not organically connected, through established ties of kinship, locality, and occupation, with any single one.

What assumptions can be made about the stranger from the passage above?

(A)  The stranger can become an owner of soil through developing all kinds of charm in more intimate relations.

(B)  The stranger cannot become an owner of soil either in the physical or psychological sense.

(C)  The stranger can become an owner of soil through establishing ties of kinship and so on.

(D)  The stranger might become an owner of soil in the physical sense but not in the psychological

Answer: (B)

22. L is the only son of A and S. S has one sibling, B, who is married to L’s aunt, K. B is the only son of D. How are L and D related?

Select the possible option(s):

(A)  Grandchild and Paternal Grandfather

(B)  Grandchild and Maternal Grandfather

(C)  Grandchild and Paternal Grandmother

(D)  Grandchild and Maternal Grandmother

Answer: (B, D)

23. Five segments of a sentence are given below. The first and fifth segments are at their correct positions, while the middle three segments (represented by 2, 3, and 4) are jumbled up. Choose the correct order of the segments so that they form a coherent sentence:

(1) Consumed multitudes are jostling and shoving inside me

(2) and guided only by the memory of a large white bedsheet with a roughly circular hole some seven inches in diameter cut into the center,

(3) clutching at the dream of that holey, mutilated square of linen, which is my talisman, my open-sesame,

(4) I must commence the business of remaking my life from the point at which it really began,

(5) some thirty-two years before anything as obvious, as present, as my clock-ridden, crime-stained birth.

(A)  2 – 3 – 4

(B)  3 – 2– 4

(C)  4 – 2– 3

(D)  4 – 3 – 2

Answer: (A)

24. “I told you the truth,” I say yet again, “Memory’s truth, because memory has its own special kind. It selects, eliminates, alters, exaggerates, minimizes, glorifies, and vilifies also; but in the end it creates its own reality, its heterogeneous but usually coherent versions of events; and no sane human being ever trusts someone else’s version more than his own.”

What are the different ways in which ‘truth’ can be understood from the passage?

(A)  Truth is what can be verified by hard empirical evidence.

(B)  Truth is based on what can be perceived by the senses.

(C)  Truth is the product of memory that is fallible, selective and slanted.

(D)  Truth is contingent on the observer and can only be partial.

Answer: (C, D)

25. A firm needs both skilled labour and unskilled labour for the production of cloth. The wage of skilled labour is Rs. 40,000 per month, and that of unskilled labour is Rs. 15,000 per month. The total wage bill of the firm for the production of cloth is Rs. 23,75,000 in a month for 100 labour. How many skilled labour are employed by the firm (in Integer)?

Answer: (35 to 35)

26. Select the odd word and write the option number as answer:

(1) Lek (2) Zloty (3) Diner (4) Drachma (5) Real

Answer: (3 to 3)

Sociology – C6

XH-C6: Q.27 – Q.44 Carry ONE mark Each

27. ___________ has given the concept of ‘thick description’.

(A)  Emile Durkheim

(B)  Clifford Geertz

(C)  Louis Dumont

(D)  Talcott Parsons

Answer: (B)

28. Objectivity in social science research as a matter of transpersonal replicability entails ____________.

(A)  Merciless elimination of personal biases

(B)  Allocation of costs and benefits to parties be made on normative standards

(C)  Basic honesty in application of professional norms of research

(D)  Reproducibility of findings of research under similar conditions and methods

Answer: (D)

29. Which of the following is NOT a characteristic of hypothesis?

(A)  A shrewd hunch suggestive of a possible solution to a problem

(B)  A causal relationship between two/more variables

(C)  An empirical conclusion foregone

(D)  A theoretically derived statement that denies refutability

Answer: (D)

30. ___________ theory most appropriately describes a hierarchy of wealthy ‘core’ nations, poor ‘periphery’ nations, and a middle group of ‘semi-periphery’ nations.

(A)  Globalization

(B)  Stages of growth

(C)  World systems

(D)  Limits to growth

Answer: (C)

31. In Hindu society, marriage of a widow to the husband’s brother is referred to as __________.

(A)  Polygyny

(B)  Endogamy

(C)  Levirate

(D)  Polyandry

Answer: (C)

32. Because of Covid-19 lockdown, a large number of working-class people lost their jobs. Rising price of commodities has also worsened their economic conditions. ___________ concept of Karl Marx most appropriately describes this ‘increasing impoverishment’ of the poor in contemporary times.

(A)  Alienation

(B)  Commodity fetishism

(C)  Pauperization

(D)  Embourgeoisement

Answer: (C)

33. ___________ coined the term, ‘ethnocentrism’.

(A)  A.R. Radcliffe-Brown

(B)  Bronislaw Malinowski

(C)  W.G. Sumner

(D)  Harold Garfinkel

Answer: (C)

34. Supremacy of science over non-sciences is attributed to ___________.

(A)  Postcolonialism

(B)  Neo-Kantianism

(C)  Positivism

(D)  Verstehen

Answer: (C)

35. Communism, universalism, disinterestedness and ______________ constitute Mertonian ethos of science.

(A)  Organized skepticism

(B)  Organized dogmatism

(C)  Objectivity

(D)  Neutrality

Answer: (A)

36. ____________theory postulates that developing economies would eventually catch up with the developed economies if they follow the social and economic models of Western capitalism.

(A)  Postmodern

(B)  Subaltern

(C)  Marxist

(D)  Modernization

Answer: (D)

37. ____________ approach was NOT propounded by B.S. Cohn as one of the approaches to study Indian civilization.

(A)  Administrative

(B)  Missionary

(C)  Orientalist

(D)  Historiographical

Answer: (D)

38. If standard deviation is 0.3, then its corresponding variance would be _________.

(A)  0.9

(B)  0.03

(C)  0.09

(D)  0.3

Answer: (C)

39. Under __________ land tenurial system during the British rule in India, the individual cultivator/peasant had some ownership right over land holding.

(A)  Zamindari

(B)  Ryotwari

(C)  Mahalwari

(D)  Talukdari

Answer: (B)

40. ____________ introduced the concept of ‘reconstructive science’.

(A)  J. Habermas

(B)  A. Giddens

(C)  P. Bourdieu

(D)  M. Foucault

Answer: (A)

41. _____________ theory lies between minor working hypotheses and master conceptual schemes.

(A)  Middle-range

(B)  Social action

(C)  Conflict

(D)  Symbolic interactionist

Answer: (A)

42. __________ is NOT a characteristic feature of M.N. Srinivas’s concept of ‘dominant caste’.

(A)  Numerical strength

(B)  Economic power through possession of land

(C)  Political power

(D)  Membership in militant organizations

Answer: (D)

43. Max Weber’s instrumental rationality refers to ___________.

(A)  Traditional action

(B)  Emotive action

(C)  Goal-rational action

(D)  Value-rational action

Answer: (C)

44. The stable pattern of ‘modern men’ formulated by Alex Inkeles does not include __________.

(A)  Openness to new experiences

(B)  Freedom from traditional authority

(C)  Rejection of activities in civil politics

(D)  Belief in science and technology

Answer: (C)

XH-C6: Q.45 – Q.65 Carry TWO marks Each

45. _____________ refers to the careful consideration of the ways in which researchers’ past experiences, points of view, and roles impact these same researchers’ interactions with, and interpretations of, the research scene.

(A)  Self-reflexivity

(B)  Social interactionism

(C)  Objectivity

(D)  Participant action research

Answer: (A)

46. __________ introduced the concept of __________ to refer to situations in which people accept, consent to, internalize, and are complicit in reproducing values and norms that are not in their own best interests.

(A)  K. Marx, alienation

(B)  M. Foucault, governmentality

(C)  A. Gramsci, hegemony

(D)  R. Putnam, social capital

Answer: (C)

47. ___________ is the process by which researchers begin by identifying several participants who fit the study’s criteria and then ask these people to suggest a colleague, a friend, or a family member who also fits the study’s criteria.

(A)  Random sampling

(B)  Stratified random sampling

(C)  Purposive sampling

(D)  Snowball sampling

Answer: (D)

48. Match the following sociologists as given in Column P with their views on religion as given in Column Q.

(A)  A-I, B-II, C-V, D-III

(B)  A-I, B-III, C-V, D-II

(C)  A-IV, B-III, C-II, D-V

(D)  A-I, B-III, C-IV, D-II

Answer: (D)

49. There are four forms of triangulation, namely data triangulation, __________, theory triangulation and _____________.

(A)  Field triangulation, methodological triangulation

(B)  Investigator triangulation, methodological triangulation

(C)  Investigator triangulation, interviewee triangulation

(D)  Investigator triangulation, epistemological triangulation

Answer: (B)

50. ____________ is the process by which groups seek to preserve some advantage by monopolizing resources and restricting access to the group.

(A)  Social action

(B)  Social construction

(C)  Social closure

(D)  Social conflict

Answer: (C)

51. Correlation coefficient lies between __________ and __________, and probability lies between _________ and __________.

(A)  – 1 and 1, 0 and 1

(B)  0 and 1, – 1 and 1

(C)  – 0.5 and 1.5, 0 and 1

(D)  – 1 and 1, – 0.5 and 1.5

Answer: (A)

52. Match the following concepts as given in Column P with the Sociologists as given in Column Q:

(A)  A-III, B-IV, C-I, D-II

(B)  A-III, B-I, C-IV, D-II

(C)  A-III, B-II, C-IV, D-I

(D)  A-IV, B-III, C-II, D-I

Answer: (A)

53. Match the following approaches and village study as given in Column P with Sociologists as given in Column Q:

(A)  A-III, B-IV, C-I, D-II

(B)  A-II, B-IV, C-III D-I

(C)  A-II, B-IV, C-I, D-III

(D)  A-IV, B-III, C-II, D-I

Answer: (B)

54. According to the World Commission on Environment and Development, ‘sustainable development’ refers to ______________.

(A)  Development that meets the needs of the present

(B)  Development that meets the needs of the future

(C)  Development that meets the needs of the present without compromising the ability of future generations to meet their own needs

(D)  Development that meets the needs of the past

Answer: (C)

55. According to David Pocock, the Indian Jajmani system reflects ___________.

(A)  Hierarchical collectivity

(B)  Regulated individuality

(C)  Egalitarian collectivity

(D)  Anarchic individuality

Answer: (B, C)

56. Which of the following is/are NOT characteristic(s) of class system?

(A)  Class systems are fluid and the boundaries between classes are never clear-cut.

(B)  Class is a form of stratification in which one’s social position is given for a lifetime.

(C)  Class is economically based.

(D)  Class divisions are organized around the purity and pollution.

Answer: (B, D)

57. Which of the following concept(s) is/are NOT propounded by Emile Durkheim?

(A)  Collective bargaining

(B)  Collective effervescence

(C)  Cult of the individual

(D)  Charismatic authority

Answer: (A, D)

58. The example(s) of relations of production is/are ___________.

(A)  Labour

(B)  Division of labour

(C)  Property relations

(D)  Capitalism

Answer: (B, C)

59. While __________ emerges around a charismatic figure, idea or vision, __________ emerges as a breakaway group from a preexisting body of belief, rituals, or believers.

(A)  Cult, Sect

(B)  Sect, Cult

(C)  Totem, Cult

(D)  Religion, Sect

Answer: (A)

60. Which of the following statement(s) is/are true for the Forest Rights Act (2006) of India?

(A)  Forest dwellers’ right of conserving and protecting community forest resources

(B)  Forest dwellers’ right to rehabilitation

(C)  Forest dwellers’ right to settlement from forest villages into revenue villages

(D)  Forest dwellers’ absolute right over all the natural resources within the given forest area

Answer: (A, B, C)

61. The Ethnographic Survey of India as part of the Census 1901 was conducted to __________.

(A)  Mitigate the racial and cultural issues faced by the British

(B)  Make suitable legislations for Indian subjects

(C)  Protect the primitive beliefs and usages of Indians

(D)  Take executive action (welfare and development)

Answer: (A)

62. ____________ and ___________ propounded conflict theory.

(A)  Talcott Parsons

(B)  Ralf Dahrendorf

(C)  Lewis A. Coser

(D)  Jeffrey C. Alexander

Answer: (B, C)

63. Which of the following is/are NOT aspect(s) of secularization process?

(A)  Privatization of religion

(B)  Rising level of membership of religious organizations

(C)  Loss of social and political influence of religious organizations

(D)  Belief in magic and supernatural forces

Answer: (B, D)

64. Which of the following pair(s) is/are correct?

M1: Michel Foucault M2: Pierre Bourdieu M3: John Urry

X1: Archaeology of Knowledge X2: Sociological Imagination X3: Practice Theory

(A)  M1 – X1

(B)  M2 – X2

(C)  M3 – X2

(D)  M2 – X3

Answer: (A, D)

65. Which of the following statement(s) is/are INCORRECT?

(A)  Hypergamy is a form of marriage where bride’s family is of superior status to the groom’s family

(B)  Hypogamy is a form of marriage where groom’s family is of superior status to the bride’s family

(C)  Exogamy refers to marriage outside the caste or sub-caste group

(D)  Polygynandry refers to marriage of several men to several women at the same time

Answer: (A, B)

Latest Govt Job & Exam Updates:

View Full List ...

© Copyright Entrance India - Engineering and Medical Entrance Exams in India | Website Maintained by Firewall Firm - IT Monteur